PDA

نسخه کامل مشاهده نسخه کامل : ◄◄ اتــاق تــجــزیــه و تــرکــیــبــات ►►



صفحه ها : [1] 2 3

Ali_p30
28-04-2006, 19:38
هركسي اطلاعاتي درباره دو جمله اي نيوتوني داره بگه ممنون ميشم.

Hidden-H
28-04-2006, 19:40
سلام
من نفهميدم منظورتونو
اگه مي شه بيشتر بگيد شايد تونستم كمك كنم
ببخشيد
ممنون

Ali_p30
28-04-2006, 19:51
اتحاد ها رو احتمالا بايد خونده باشيد.
اين براي توان 5 به بالا استفاده ميشه.

Hidden-H
28-04-2006, 20:23
براي توان 5 به بالا؟
من شرمنده

M E H D I
29-04-2006, 23:35
يه چيزي بود بسط مي گفتن بهش. از مثلث خيام پاسكال. منظورت اونه؟ [ برای مشاهده لینک ، لطفا با نام کاربری خود وارد شوید یا ثبت نام کنید ]

Ali_p30
30-04-2006, 00:11
فكر نكنم! حالا هميني كه گفتي رو يه خورده توضيح بده
ممنون

hadian123
31-05-2006, 12:46
دوست عزيز دو قا نون نيوتن به شرح زير است :

1- هرگاه نيرويي بر جسمي بر خلاف ميدان نشست وارد شود آن جسم با نيروي ثانويه آن حركت به جلو مي رود.
2-هرگاه به جسمي نيرويي وارد شود آننيرو با چهار نيروي ديگر ارتباط برقرار كرده و گرايش به برگشت را با فشار بيش از حد معمول دارا مي باشد.

Monica
31-05-2006, 13:36
دوست عزيز دو قا نون نيوتن به شرح زير است :

1- هرگاه نيرويي بر جسمي بر خلاف ميدان نشست وارد شود آن جسم با نيروي ثانويه آن حركت به جلو مي رود.
2-هرگاه به جسمي نيرويي وارد شود آننيرو با چهار نيروي ديگر ارتباط برقرار كرده و گرايش به برگشت را با فشار بيش از حد معمول دارا مي باشد.
آقا رامبد کجایی که واسه تاپیکت سوژه ی بدون شرح یافتم :biggrin:
دوست عزیز منظور ایشون قوانین نیوتون نبود.
( امیدوارم از شوخی من ناراحت نشده باشید :happy: )

يه چيزي بود بسط مي گفتن بهش. از مثلث خيام پاسكال. منظورت اونه؟
اگه منظورتون اینه بگید تا براتون بذارمش.
اگه نه همین جوری الکی نگید بذار آخه تایپش یه کم سخته همش توان داره :biggrin:
MerC & ByE [ برای مشاهده لینک ، لطفا با نام کاربری خود وارد شوید یا ثبت نام کنید ]

mska
31-05-2006, 13:54
فكر كنم منظور شما اين باشه
ايشاالله كه درسته
زياد يادم نيست فكر كنم مربوط به رياضي 1 كه خيلي وقت پيش پاس شد.
[ برای مشاهده لینک ، لطفا با نام کاربری خود وارد شوید یا ثبت نام کنید ]

M E H D I
02-06-2006, 01:02
سلام

شرمنده كه دير دارم جواب ميدم. من چند روز بود تو اين انجمن نمي اومدم.

مثلث خيام پاسكال اينه:


[ برای مشاهده لینک ، لطفا با نام کاربری خود وارد شوید یا ثبت نام کنید ]

(هر عدد از جمع دو عدد بالاييش به دست مياد)

براي مثال من چند تاش رو مي نويسم:


[ برای مشاهده لینک ، لطفا با نام کاربری خود وارد شوید یا ثبت نام کنید ]

اگه متوجه نشدين بگين تا بيشتر توضيح بدم. :happy:

soleares
29-09-2006, 21:43
آنالیز نام عمومی آن بخش‌هائی از ریاضیات است که با مفاهیم حد و همگرایی مربوط‌اند و در آن‌ها موضوعاتی مثل پیوستگی و انتگرال‌گیری و مشتق‌پذیری و توابع غیرجبری بررسی می‌شود. این موضوعات را معمولاً در عرصه اعداد حقیقی یا اعداد مختلط و توابع مربوط به آن‌ها بحث می‌کنند ولی می‌توان آنها را در هر فضائی از موجودات ریاضی که در آن مفهوم "نزدیکی" (فضای توپولوژیک) یا "فاصله" (فضای متریک) وجود دارد به‌کار برد. آنالیز ریاضی از کوشش‌های مربوط به دقیق کردن مبانی و تعریف‌های حسابان سر برآورده است.

soleares
29-09-2006, 21:46
آنالیز عددی الگوریتم حل مسئله در ریاضیات پیوسته (ریاضیاتی که جدا از ریاضیات گسسته است)را مورد مطالعه قرار می‌دهد. آنالیز عددی اساسا به مسائل مربوط به متغیرهای حقیقی و متغیرهای مختلط و نیز جبر خطی عددی به علاوه حل معادلات دیفرانسیل و دیگر مسائلی که از فیزیک و مهندسی مشتق می‌شود.

معرفی
تعدادی از مسائل در ریاضیات پیوسته دقیقا با یک الگوریتم حل می‌شوند.که به روش‌های مستقیم حل مسئله معروف اند.برای مثال روش حذف گائوسی برای حل دستگاه معادلات خطی است و نیز روش سیمپلکس در برنامه ریزی خطی مورد استفاده قرار می‌گیرد. ولی روش مستقیم برای حل خیلی از مسائل وجود ندارد.و ممکن است از روشهای دیگر مانند روش تکرارشونده استفاده شود،چون این روش می‌تواند در یافتن جواب مسئله موثرتر باشد.

برآورد خطاها
تخمین خطاهای موجود در حل مسائل از مهم‌ترین قسمت‌های آنالیز عددی است این خطاها در روش‌های تکرار شونده وجود دارد چون به هرحال جوابهای تقریبی بدست آمده با جواب دقیق مسئله، اختلاف دارد و یا وقتی که از روش‌های مستقیم برای حل مسئله استفاده می‌شود خطاهایی ناشی از گرد کردن اعداد بوجود می‌آید. در آنالیز عددی می‌توان مقدار خطا را درآخر روش که برای حل مسئله به کار می‌رود، تخمین زد.


کاربردها
الگوریتم‌های موجود در آنالیز عددی برای حل بسیاری از مسائل موجود در علوم پایه و رشته‌های مهندسی مورد استفاده قرار می‌گیرند. برای مثال از این الگوریتم‌ها در طراحی بناهایی مانند پل ها، در طراحی هواپیما، در پیش بینی آب و هوا، تهیه نقشه‌های جوی از زمین، تجزیه و تحلیل ساختار مولکول ها، پیدا کردن مخازن نفت، استفاده می‌شود، همچنین اکثر ابر رایانه‌ها به طور مداوم بر اساس الگوریتم‌های آنالیز عددی برنامه ریزی می‌شوند. به طور کلی آنالیز عددی از نتایج عملی حاصل از اجرای محاسبات برای پیدا کردن روش‌های جدید برای تجزیه و تحلیل مسائل، استفاده می‌کند.


نرم افزار‌ها
امروزه بیشتر الگوریتم‌ها توسط رایانه اجرا می‌شوند نرم افزارهایی برای اجرای محاسبات ریاضی طراحی شده اند. از مهم‌ترین و کاربردی‌ترین آنها می‌توان به نرم افزارهایی زیر اشاره کرد:




مپل (Maple)
متمتیکا (Mathematica)
جی‌ان‌یو اکتاو (GNU Octave)
متلب (Matlab)
سایلب (Scilab)
زبان برنامه‌نویسی آی‌دی‌ال (IDL)
زبان برنامه‌نویسی آر (R)

××با تشكر سولئارس××

soleares
29-09-2006, 21:50
آنالیز مختلط یا نظریه توابع، نام مبحثی در ریاضیات است که خود را با توابع مشتق‌پذیر با مقادیر مختلط مشغول می‌کند.

تابع مختلط
تابعی است که هم دامنه تعریف آن و هم مقدار آن هردو مختلط باشند. به این ترتیب، یک تابع مختلط، تابعی از فضای R2 به فضای R2 می‌باشد.
(منظور آر به توان 2 مي باشد)

مشتق‌پذیری
به تابعی که مختلط مشتق‌پذیر باشد، تابع تحليلی یا تابع تمامريخت گفته می‌شود و آن زمانی است که حد زیر در دایره بازی، در اطراف نقطه z0 وجود داشته باشد. در اینجا مسلما z یک مقدار مختلط است.


[ برای مشاهده لینک ، لطفا با نام کاربری خود وارد شوید یا ثبت نام کنید ]

تعریف بالا، هم ارز است با شرايط کوشی-ریمان که به راحتی از آن به دست می‌آید.


[ برای مشاهده لینک ، لطفا با نام کاربری خود وارد شوید یا ثبت نام کنید ]


[ برای مشاهده لینک ، لطفا با نام کاربری خود وارد شوید یا ثبت نام کنید ]

فرمول کوشی
فرمول انتگرال کوشی یا به طور بهتر قضیه کوشی، برای هر تابعی که بر روی محیط خاصی تحليلی باشد، صادق است:


[ برای مشاهده لینک ، لطفا با نام کاربری خود وارد شوید یا ثبت نام کنید ]

در اینجا، انتگرال مسیری، بر روی محیطی انجام می‌پذیرد که تابع در آن مشتق‌پذیر است.

بسط دادن
بر خلاف، توابع حقیقی، بسط تیلور برای توابع تحليلی، همیشه امکان‌پذیر است. از این گذشته، در شرایط خاصی نیز می‌توان از بسط لورنتس در این تئوری استفاده کرد.

منابع
Needham T., Visual Complex Analysis (Oxford, 1997).
Henrici P., Applied and Computational Complex Analysis (Wiley). [Three volumes: 1974, 1977, 1986.]

××ارادتمند شما سولئارس××

soleares
29-09-2006, 21:52
و اما سوالي دارم درباره آناليز هاي زير اگر اطلاعاتي داريد لطف كنيد بزاريد اينجا..

آنالیز تابعی
آنالیز ترکیبی
آنالیز تصادفی
آنالیز حقیقی
آنالیز هارمونیک

منتظر كمك هاي شما هم مي مانم .

soleares
01-10-2006, 21:51
در ریاضیات، یک تابع رابطه‌ای است که هر متغیر دریافتی خود را به فقط یک خروجی نسبت می‌دهد. علامت استاندارد خروجی یک تابع f به همراه ورودی آن، x می‌باشد یعنی‎ f(x)‏. به مجموعه ورودی‌هایی که یک تابع می‌تواند داشته باشد دامنه و به مجموعه خروجی‌هایی که تابع می‌دهد برد می‌گویند.

برای مثال عبارت f(x) = x2 نشان دهنده یک تابع است، که در آن f مقدار x را دریافت می‌کند و x2 را می‌دهد. در این صورت برای ورودی 3 مقدار 9 به دست می‌آید. برای مثال، برای یک مقدار تعریف شده در تابع f می‌توانیم بنویسیم، f(4) = 16.

معمولاً در تمارین ریاضی برای معرفی کردن یک تابع از کلمه f استفاده می‌کنیم و در پاراگراف بعد تعریف تابع یعنی f(x) = 2x+1 را می‌نویسم و سپس f(4) = 9. وقتی که نامی برای تابع نیاز نباشد اغلب از عبارت y=x2 استفاده می‌شود.

وقتی که یک تابع را تعریف می‌کنیم، می‌توانیم خودمان نامی به آن بدهیم، برای مثال:

.

یکی از خواص تابع این است که برای هر مقدار باید یک جواب وجود داشته باشد، برای مثال عبارت:



یک تابع نمی‌باشد، زیرا ممکن است برای یک مقدار دو جواب وجود داشته باشد. جذر عدد 9 برابر 3 است و در این رابطه اعداد +3 و -3 به دست می‌آیند. برای ساختن یک تابع ریشه دوم، باید فقط یک جواب برای آن وجود داشته باشد، یعنی:

,

که برای هر متغیر غیرمنفی یک جواب غیرمنفی وجود دارد.

در یک تابع لزومی ندارد که حتماً بر روی عدد علمیاتی انجام گیرد. یک مثال که نشان می‌دهد که عملیاتی بر روی عدد انجام نمی‌شود، تابعی است که پایتخت یک کشور را معین می‌کند. مثلاً Capital(France) = Paris.

حال کمی دقیق‌تر می‌شویم اما هنوز از مثال‌های خودمانی استفاده می‌کنیم. A و B دو مجموعه هستند. یک تابع از A به B با به هم پیوستن مقادیر منحصر به فرد درون A معین می‌شود و مجموعه B به دست می‌آید. به مجموعه A دامنه تابع می‌گویند؛ مجموعه B هم تمام مقادیری را که تابع می‌تواند داشته باشد شامل می‌شود.

در بیشتر زمینه‌های ریاضی، اصطلاحات تبدیل و نگاشت معمولاً با تابع هم معنی پنداشته می‌شوند. در هر حال ممکن است که در بعضی زمینه‌های خصوصیات دیگری داشته باشند. برای مثال در هندسه، یک نگاشت گاهی اوقات یک تابع پیوسته تعریف می‌شود.


تعاریف ریاضی یک تابع
یک تابع f یک رابطه دوتایی است، به طوری که برای هر x یک و فقط یک y وجود داشته باشد تا x را به y رابطه دهد. مقدار تعریف شده و منحصر به فرد y با عبارت (f(x نشان داده می‌شود.

به دلیل اینکه دو تعریف برای رابطه دوتایی استفاده می‌شود، ما هم از دوتعریف برای تابع استفاده می‌کنیم.


تعریف اول
ساده تعریف رابطه دوتایی عبارتست از: «یک رابطه دوتایی یک زوج مرتب می‌باشد». در این تعریف اگر رابطه دوتایی دلالت بر «کوچکتر از» داشته باشد آن گاه شامل زوج مرتب‌هایی مانند (2, 5) است، چون 2 از 5 کوچکتر است.

یک تابع مجموعه‌ای از زوج مرتب‌ها است به طوری که اگر (a,b) و (a,c) عضوی از این مجموعه باشند آن گاه b با c برابر باشد. در این صورن تابع مجذور شامل زوج (3, 9) است. رابطه جذر یک تابع نمی‌باشد زیرا این رابطه شامل زوج‌های (9, 3) و (9, -3) است و در این صورت 3 با -3 برابر نیست.

دامنه تابع مجموعه مقادیر x یعنی مختص‌های اول زوج‌های رابطه مورد نظر است. اگر x در دامنه تابع نباشد آن گاه (f(x هم تعریف نشده‌است.

برد تابع مجموعه مقادیر y یعنی مختص‌های دوم زوج‌های رابطه مورد نظر است.


تعریف دوم
بعضی از نویسندگان نیاز به تعریفی دارند که فقط از زوج‌های مرتب استفاده نکند بلکه از دامنه و برد در تعریف استفاده شود. این گونه نویسندگان به جای تعریف زوج مرتب از سه‌تایی مرتب (X,Y,G) استفاده می‌کنند، که در آن X و Y مجموعه هستند (که به آنها دامنه و برد رابطه می‌گوییم) و G هم زیرمجموعه‌ای از حاصل‌ضرب دکارتی X و Y است (که به آن گراف رابطه می‌گویند). در این صورت تابع رابطه دوتایی است که در آن مقادیر X فقط یک بار در اولین مختص مقادیر G اتفاق می‌افتد. در این تعریف تابع دارای برد منحصر به فرد است؛ این خاصیت در تعریف نخست وجود نداشت.

شکل تعریف تابع بستگی به مبحث مورد نظر دارد، برای مثال تعریف یک تابع پوشا بدون مشخص کردن برد آن امکان‌ناپذیر است.


پیشینه تابع
«تابع»، به عنوان تعریفی در ریاضیات، توسط گاتفرید لایبنیز در سال 1694، با هدف توصیف یک کمیت در رابطه با یک منحنی به وجود آمد، مانند شیب یک نمودار در یک نقطه خاص. امروزه به توابعی که توسط لایبنیز تعریف شدند، توابع مشتق‌پذیر می‌گوییم، اغلب افراد این توابع در هنگام آموختن ریاضی با این گونه توابع برمی خورند. در این گونه توابع افراد می‌توانند در مورد حد و مشتق صحبت کنند. چنین توابعی پایه حسابان را می‌سازند.

واژه تابع بعدها توسط لئونارد اویلر در قرن هجدهم، برای توصیف یک عبارت یا فرمول شامل متغیرهای گوناگون مورد استفاده قرار گرفت، مانند f(x) = sin(x) + x3.

در طی قرن نوزدهم، ریاضی‌دانان شروع به فرموله کردن تمام شاخه‌های ریاضی کردند. ویرسترس بیشتر خواهان به وجود آمدن حسابان در علم حساب بود تا در هندسه، یعنی بیشتر طرفدار تعریف اویلر بود.

در ابتدا، ایده تابع ترجیحاً محدود شد. برای ژوزف فوریه مدعی بود که تمام توابع از سری فوریه پیروی می‌کنند در حالی که امروزه هیچ ریاضی‌دانی این مطلب را قبول ندارد. با گسترش تعریف توابع، ریاضی‌دانان توانستند به مطالعه «عجایب» در ریاضی بپردازند از جمله این که یک تابع پیوسته در هیچ مکان گسستنی نیست. این توابع در ابتدا بیان نظریه‌هایی از روی کنجکاوی فرض می‌شد و آنها از این توابع برای خود یک «غول» ساخته بودند و این امر تا قرن بیستم ادامه داشت.

تا انتهای قرن نوزدهم ریاضی‌دانان سعی کردند که مباحث ریاضی را با استفاده از نظریه مجموعه فرموله کنند و آنها در هر موضوع ریاضی به دنبال تعریفی بودند که از مجموعه استفاده کند. دیریکله و لوباچوسکی هر یک به طور مستقل و تصادفاً هم زمان تعریف «رسمی» از تابع دادند.

در این تعریف، یک تابع حالت خاصی از یک رابطه است که در آن برای هر مقدار اولیه یک مقدار ثانویه منحصر به فرد وجود دارد.

تعریف تابع در علم رایانه، به عنوان حالت خاصی از یک رابطه، به طور گسترده‌تر در منطق و علم تئوری رایانه مطالعه می‌شود.

soleares
01-10-2006, 21:53
توابع مورد استفاده در اکثر علوم کمی می‌باشند، برای مثال در فیزیک، هنگامی که می‌خواهیم رابطه بین چند متغیر را بیان کنیم، مخصوصاً در زمانی که مقدار یک متغیر کاملاً وابسته به متغیر دیگر است. برای مثال وقتی که می‌خواهیم نشان دهیم که تغییر دمای آب چه تاثیری بر روی چگالی آن می‌گذارد.

توابع را همچنین مورد استفاده در علم رایانه برای مدل‌سازی ساختمان داده‌ها و تاثیرات الگوریتم می‌بینیم. این کلمه در رویه‌ها و زیرروال‌ها بسیار دیده می‌شود.

به یک مقدار ورودی مشخص در یک تابع، آرگومان تابع می‌گویند. برای هر آرگومان x، مقدار منحصر به فرد y در مجموعه اعداد برد تابع وجود دارد که با آن مطابقت می‌کند، و به آن مقدار در x یا تصویر x تحت f می‌گویند. تصویر x می‌تواند با (f(x و یا y نشان داده شود.

گراف تابع f مجموعه تمام زوج مرتب‌های ((x, f(x) به ازای تمام xهای درون دامنه X است. اگر X و Y زیرمجموعه‌هایی از R (اعداد حقیقی) باشند، در این صورت این تعریف مانند شهود «گراف» به عنوان یک تصویر یا نمودار تابع به همراه زوج مرتب‌های نقاط در محور مختصات است.

مفهوم تصویر را می‌توان اتصال مجموعه‌ای از نقاط تصویر به هم دانست. اگر A زیرمجموعه‌ای دامنه باشد، آن گاه (f(A هم زیرمجموعه‌ای از برد است که شامل تمام تصویرهای‌های مقادیر A می‌شود. در این صورت می‌گوییم که (f(A تصویر A تحت f است.

به یاد داشته باشید که برد f همان تصویر (f(X در مقادیر دامنه‌است و برد f زیرمجموعه‌ای از مجموعه تمام مقادیر ممکن برای f است.

وارون (یا معکوس) مجموعه B که مجموعه مقایر ممکن برای Y تحت تابع f است زیرمجموعه‌ای از دامنه X است که به این صورت تعریف می‌شود:

f −1(B) = {x in X | f(x) is in B}
برای مثال، وارون مجموعه {4, 9} تحت تابع مربع مجموعه {−3,−2,+2,+3} است.

به طور کلی، وارون یک نقطه منحصر به فرد (نقطه‌ای که فقط یک مقدار برای آن وجود داشته باشد)، می‌تواند مجموعه تمام اعداد را دربرگیرد. برای مثال اگر f(x) = 7 باشد، آن گاه وارون {5} تهی است اما وارون {7} برابر مقادیر دامنه آن است. در این صورت وارون یک مقدار در برد زیرمجموعه‌ای از دامنه آن است. طبق قرارداد وارون یک مقدار یعنی f −1(b) ویا همان f −1({b}) به صورت زیر است:

f −1(b) = {x in X | f(x) = b}

مهمترین توابع عبارتند از:

تابع یک‌به‌یک، که در آن این خاصیت وجود دارد که اگر f(a) = f(b) باشد آن گاه a هم باید با b برابر باشد.
تابع پوشا، که در آن این خاصیت وجود دارد که برای هر y در برد، یک مقدار x در دامنه وجود داشته باشد یعنی f(x) = y.
توابعی که هم یک‌به‌یک و هم پوشا هستند.
اگر از تعریف اول تابع که در بالا گفته شد استفاده شود، تا موقعی که برد تعریف نشده باشد، «یک‌به‌یک» بودن تابع باید وضعیتی مانند پوشا بودن را داشته باشد. می‌توان از ترکیب دو یا چند تابع به عنوان یک تابع استفاده کرد. برای مثال، f(x) = sin(x2) ترکیب یک تابع سینوسی و یک تابع درجه دو است. توابع f: X → Y و g: Y → Z می‌توانند با هم ترکیب شوند، به طوری که ابتدا این عمل بر روی تابع f انجام شود و y = f(x) به دست آید و یک بار هم بر روی g اعمال شود و z = g(y) به دست آید. تابع مرکب g و f به صورت زیر نوشته می‌شود:



ابتدا تابع سمت راست عملیات را انجام می‌دهد و سپس تابع سمت چپ (برعکس زبان انگلیسی) و این تابع را «جی‌اُاِف» می‌خوانیم.

در تعریفی غیرعلمی، تابع وارون f تابعی است که اثر تابع f را خنثی کند، به این صورت که هر مقدار (f(x را به آرگومان x نسبت دهد. تابع مربع (درجه دو) وارون تابع غیرمنفی جذر (ریشه دوم) است، به طوری که اگر f دارای دامنه X و برد Y و گراف G باشد، آن گاه وارون آن دارای دامنه Y و برد X و گراف است.

G−1 = { (y, x) : (x, y) ∈ G }

برای مثال اگر گراف f برابر G = {(1,5), (2,4), (3,5)} باشد، آن گاه گراف f−1 برابر G−1 = {(5,1), (4,2), (5,3)} می‌شود.

رابطه f−1 یک تابع است اگر و تنها اگر برای هر y در برد فقط یک آرگومان x مانند f(x) = y وجود داشته باشد، به عبارت دیگر، وارون تابع f یک تابع است اگر و تنها اگر f پوشا و یک‌به‌یک باشد. در این مثال، برای هر x درون X f−1(f(x)) = x و برای هر y درون Y f(f−1(y)) = y است. گاهی اوقات می‌توان یک تابع را تغییر داد و این کار اغلب با جایگذاری دامنه‌ای جدید که زیرمجموعه‌ای از دامنه قبلی باشد صورت می‌گیرد، و همینطور باید تغییرات را در برد و گراف اعمال کرد که در این صورت تابع تغییر داده شده دارای وارونی است که خود یک تابع است.

برای مثال وارون تابع y = sin(x)، یعنی f(x) = arcsin (x)، به صورت y = arcsin (x) تعریف می‌شود اگر و تنها اگر x = sin(y) باشد، و این یک تابع نیست زیرا گراف آن شامل دو زوج مرتب (0, 0) و (0, 2π) است. اما اگر دامنه y = sin(x) را به −π/2 ≤ x ≤ π/2 تغییر دهیم، برای برد داریم −1 ≤ y ≤ 1 و در این صورت وارون تابع مورد نظر یک تابع است، و برای بیان آن از A در حرف اول آن استفاده می‌کنیم، یعنی (f(x) = Arcsin (x.

اما این روش برای همه توابع عملی نیست، زیرا در بعضی موارد پیدا کردن وارون توابع غیرممکن است.


اگر دامنه X تعریف شده باشد، تابع f را می‌توان با جدول‌بندی کردن آرگومان‌های x و جواب آنها در f(x) تعریف کرد.

چیزی که برای تعریف کردن تابع رایج‌تر است استفاده از فرمول و به طور کلی استفاده از الگوریتم است، که در آن نشان داده می‌شود چه عملیاتی باید بر روی xهای دامنه انجام گیرد تا f(x) به دست آید. برای تعریف یک تابع می‌توان از عمل ریاضی که با آرگومان x رابطه‌ای داشته باشد استفاده کرد. البته راه‌های زیاد دیگری برای تعریف یک تابع وجود دارد؛ از جمله استفاده از روش بازگشتی، استفاده از بسط‌های تجزیه و عبارات جبری، حدها، دنباله‌ها، سری‌ها و استفاده از معادلات دیفرانسیل.

در ریاضیات توابع زیادی وجود دارد که نمی‌توانند مفهوم خود را به طور دقیق برسانند. یکی از نتایج اصلی نظریه شمارش این است که توابع زیادی وجود دارند که تعریف می‌شوند اما قابل محاسبه نیستند.

soleares
01-10-2006, 21:56
معمولاً پرانتزهای کنار آرگومان را هنگامی که برای آن ابهامی وجود ندارد حذف می‌کنند، مانند: sin x. در برخی موارد علمی، از علامت نشان‌گذاری لهستانی معکوس استفاده می‌شود، که با این کار باید پرانتزها را حذف کرد؛ و برای مثال تابع فاکتوریل همواره به صورت n! نوشته می‌شود، در حالی که اکثر افراد تابع گاما را به صورت (Γ(n می‌نویسند.

برای نشان دادن یک تابع ابتدا نام آن را می‌آوریم، سپس دامنه، بعد برد و در انتها هم ضابطه تابع را می‌نویسیم. با استفاده از این روش اغلب تابع به دو قسمت نشان داده می‌شود، مانند:



در اینجا دامنه تابع با نام «f» اعداد طبیعی و برد آن اعداد حقیقی است، و n را به خودش تقسیم بر π تبدیل می‌کند. (در بعضی موارد نام تابع را به همراه دونقطه، در بالای پیکان می‌آورند). روش نشان دادن دیگری هم وجود دارد که رایج‌تر اما غیرعلمی‌تر است، در این روش تابع به شکل کوتاه‌ شده زیر نشان داده می‌شود:



در این روش اطلاعات کمتری ارائه شده‌است و ما از دامنه و برد تابع خبر نداریم، و در این صورت به جای n می‌توانیم هر عددی از قبیل اعداد گنگ هم قرار دهیم.

بعضی از نویسندگان به جای استفاده از (f(A از [f[A استفاده می‌کنند و این کار برای رفع ابهام میان دریافت مفاهیم است، بعضی دیگر هم از f`x به جای (f(x، و f``A به جای [f[A استفاده می‌کنند.



توابع دو (یا چند) متغیره
مفهوم تابع را می‌توان با ترکیب دو یا چند آرگومان بیان کرد. این مفهوم شهودی، زمانی می‌تواند تعریف شود که دامنه تابع حاصل‌ضرب دکارتی دو یا چند مجموعه باشد.

برای مثال، عملیات را بر روی تابع ضربی که از دو عدد صحیح برای به دست آوردن حاصل استفاده می‌کند انجام می‌دهیم: f(x, y) = x·y. تابع می‌تواند دامنه Z×Z، مجموعه تمام زوج‌ها به عنوان برد Z، و برای گراف، مجموعه تمام زوج‌های ((x,y), x·y) را داشته باشد. به یاد داشته باشید که مولفه اول چنین زوج‌هایی یک زوج از اعداد صحیح است، در حالی که مولفه دوم تنها یک عدد صحیح است.

مقدار تابع زوج (x,y) برابر است با f((x,y)). اگر چه معمولاً یک جفت از پرانتزها را حذف می‌کنند و آن را به شکل f(x,y) نشان می‌دهند، یعنی یک تابع دومتغیره شامل x و y.



توابعی که حاصلشان یک مجموعه ضرب است
در این گونه توابع، مقدار تابع شامل چند متغیر است. برای مثال، تابع mirror(x, y) = (y, x) را با دامنه R×R و برد R×R در نظر بگیرید. زوج (y, x) یکی از مقادیر برد تابع که یک مجموعه‌است، می‌باشد.


عملیات دوتایی
منظور ار عملیات دوتایی ساده در ریاضی همان جمع و ضرب است، وقتی که در توابع استفاده شوند مقادیر را از Z×Z به Z می‌برند. این موضوع در جبر شرح داده می‌شود و در آنجا از توابع nتایی برای انجام عملیات استفاده می‌شود.

چیزی که از گذشته مورد استفاده قرار می‌گرفته این است که از عملیات جمع و ضرب به عنوان نشانه‌های میان‌وندی استفاده شود: x+y و x×y به جای +(x, y) و ×(x, y).

مجموعه توابع
مجموعه یک تابع از یک مجموعه X به یک مجموعه Y به صورت X → Y یا [X → Y] یا YX نشان داده می‌شود. آخرین عبارت یاد شده با استفاده از قضیه بدیهی |YX| = |Y||X| اثبات می‌شود. جزئیات بیشتر در اعداد اصلی بیان شده‌است.

معمولاً از عبارت f: X → Y برای بیان f ∈ [X → Y] استفاده می‌شود، و «f تابعی از X به Y است» خوانده می‌شود. بعضی افراد هم آن را «f: X → Y» می‌نویسند.


آیا یک تابع بیشتر از گرافش است؟
بعضی از ریاضی‌دانان از یک رابطه دوتایی (از اینجا به بعد آن را تابع می‌گوییم) به عنوان سه‌تایی مرتب (X, Y, G) استفاده می‌کنند، که در آن X و Y مجموعه دامنه و برد، و G گراف f است. اگر چه، سایر ریاضی‌دانان رابطه‌ای را تعریف می‌کنند که فقط شامل زوج‌های مجموعه G باشد، بدون این که دامنه‌ای برای آن تعیین کرده باشند.

در هر تعریف خوبی‌ها و بدی‌هایی وجود دارد، اما هر یک از آنها تعاریف مناسبی هستند که مورد استفاده در ریاضی قرار می‌گیرند. دامنه و برد موضوع مهمی است و باید به طور واضح مشخص باشند.


توابع ناقص و توابع چندتایی
وضعیت یک رابطه دوتایی f از X به Y را می‌توان به دو صورت تقسیم نمود:

f تابع جمعی: برای هر x در X، چند y در Y وجود دارد به طوری که x با y در رابطه باشد.
f تابع تک-مقداری باشد: برای هر x در X، حداقل یک y در Y وجود دارد به طوری که x با y در رابطه باشد.


در بعضی موارد که تابع وضعیت اول را دارد اما لزوماً از وضعیت دوم پیروی نمی‌کند، می‌توان تابع را تابع چندارزشی خواند؛ و رابطه‌ای که وضعیت دوم را دارد اما لزوماً از وضعیت اول پیروی نمی‌کند را می‌توان تابع ناقص خواند.

سایر توابع
توابع مختلفی وجود دارند که دارای خواصی هستند که در مباحث گوناگون ریاضی دارای اهمیت خاصی هستند. فهرستی از این توابع عبارتند از:

پیوسته، مشتق‌پذیر، انتگرال‌پذیر
خطی، چندجمله‌ای، گویا
همگرا، یکنواخت، تک‌نما
مثلثاتی
جبری
غیرجبری
منحنی
زوج، فرد
وکتور

بسط‌ها و محدودیت‌ها
در یک تعریف خودمانی، منظور از محدودیت یک تابع f، تغییر دامنه‌اش است.

اگر بخواهیم کمی دقیق‌تر نگاه کنیم، اگر f تابعی از X به Y باشد و S زیرمجموعه‌ای X، محدودیت f به S تابع f|S از S به Y می‌باشد و در این صورت می‌نویسیم برای هر s در S داریم f|S(s) = f(s).

اگر g محدودیتی از f باشد، در این صورت می‌نویسیم f بسطی از g است.

انجام عملیات در یک نقطه
اگر f: X → R و g: X → R توابعی با دامنه X و برد R باشد، آن گاه می‌توان جمع دو تابع را به این صورت تعریف کرد: f + g: X → R و توابع ضرب را هم به صورت f × g: X → R و در نتیجه:

(f + g)(x) = f(x) + g(x) (f × g)(x) = f(x) × g(x)

برای هر x در X.


توابع شمارا و غیرشمارا
تعداد کمی توابع شمارا از اعداد صحیح به اعداد صحیح وجود دارد، اما تعداد توابع از اعداد صحیح به اعداد صحیح به اندازه تعداد اعداد حقیقی است. این نشان می‌دهد که توابع از اعداد صحیح به اعداد صحیح وجود دارد که خاصیت شمارا ندارند.

soleares
01-10-2006, 21:57
کميتی که علاوه بر اندازه دارای جهت نيز باشد. مهم ترين کميت های برداری که می‌‌توان نام برد عبارت‌اند از:

۱- مکان ۲- سرعت ۳- شتاب ۴- نيرو ۵- ميدان های الکتريکی و مغناطيسی

يکی از بهترين راهای تشخيص برداری بودن يا نبودن يک کميت اينست که بررسی کنيم آيا جمع آن کميت خاصيت برداری دارد يا خير. مثلاً جريان الکتريکی با وجود آنکه علاوه بر اندازه جهت نيز دارد ولی برداری نيست زيرا جمع جريان ها به صورت اسکالر صورت می‌‌گيرد (قانون جريان کيرشهف).

در حالت بسيار کلی هر مجموعه عدد که به صورت يک ماتريس ستونی n*۱ قابل نوشتن باشد بردار گفته می‌شود. کاربرد اين مفهوم در توصيف حالت سيستم ها به مراتب بيشتر از محاسبات پديده‌های فيزيکی است.

soleares
01-10-2006, 22:00
مشتق یکی از دو مفهوم اصلی حسابان است که مقدار تغییرات لحظه‌ای تابع را نشان می‌دهد.

تعریف
مشتق تابعی مانند f، تابع 'f است که مقدارش در x با معادله‌ی زیر تعریف می‌شود:

[ برای مشاهده لینک ، لطفا با نام کاربری خود وارد شوید یا ثبت نام کنید ]

به شرطی که این حد موجود باشد.

بر طبق این تعریف مشتق مقدار تغییرات مقدار تابع است زمانی که تغییرات به صفر میل می‌کند.


نحوه‌ی نمایش
مشتق اول یک تابع تک متغیره را می‌توان به صورت‌های زیر نشان داد:

f'(x)
f(1)
[ برای مشاهده لینک ، لطفا با نام کاربری خود وارد شوید یا ثبت نام کنید ]

که این نحوه‌ی نمایش را نمایش دیفرانسیلی مشتق می‌نامند.

تاریخچه
مشتق از مسائل مهم ریاضی است که موضّع آن نیوتن و لایپ نیتز بودند و حد مقدمه آن است. نیوتن سرعت لحظه‌ای را به کمک قوانین حدگیری و لایپ نیتز شیب خط مماس بر منحنی‌ها را با استفاده از قوانین حدگیری محاسبه کرد و هر یک در حالت کلی به مشتق رسید.


مشتقات مراتب بالاتر
مشتقات مراتب بالاتر یک تابع از تعریف اصلی مشتق بدست می‌آیند. با مشتق گیری دوباره از مشتق یک تابع به مشتق دوم آن می‌رسیم و ...


نحوه‌ی نمایش
مشتقات مراتب بالاتر (مشتق مرتبه دوم، سوم و چهارم) تابع f را می‌توان به دو صورت زیر نمایش داد:

f و f و f'
f(2) و f(3) و f(4)

تابع مشتق‌پذیر در یک نقطه
اگر مشتق تابع f در نقطه‌ای مانند x موجود و معین باشد، گفته می‌شود که تابع f در نقطه‌ی x مشتق‌پذیر است.


تابع مشتق‌پذیر
اگر تابعی در هر نقطه از دامنه‌اش مشتق‌پذیر باشد، تابع مشتق‌پذیر نامیده می‌شود.


شرایط مشتق‌پذیری
برای اینکه تابعی در یک نقطه مانند x مشتق‌پذیر باشد، باید در یک همسایگی آن تعریف شده باشد و نیز در آن نقطه پیوسته باشد. یا به عبارتی تابع در آن نقطه هموار باشد.

M O H S E N
22-10-2006, 21:29
خيلي ممنون

m_honarmand_j
13-12-2006, 16:39
سلام بر همه ی دوستان
من دانش آموز سال دوم دبیرستان هستم . من سایت یا هر منبعی که در آن در مورد ترکیبیات بتوان بحث کرد سراغ ندارم . به همین جهت سعی دارم تا سؤالات خوب و جالب را در اینجا قرار بدهم تا علاقه مندان بتوانند استفاده ببرند و خود نیز ار تجربیات دیگران استفاده کنم . سعی میکنم که هر ماه حداقل 5 سؤال خوب را عنوان کنم . از دوستان خواهش می کنم اگر مطلب مناسبی دارند عنوان کنند .
باتشکر از همه ی دوستان .

m_honarmand_j
13-12-2006, 17:34
سلام
ماتریس n*n را با اعداد 0 و 1 و 1- پر می کنیم . مجموع هر سطر را مقابل آن و مجموع هر ستون را زیر آن می نویسیم . می خواهیم این 2n عدد با هم متفاوت و متوالی باشند . برای کدام n ها این کار امکان پذیر است؟

m_honarmand_j
14-12-2006, 15:49
سلام
روی هر سیاره از یک منظومه یک اختر شناس وجود دارد و هر اختر شناس نزدیکترین سیاره را مشاهده می کند . فاصله ی بین سیاره ها دو به دو متمایز است . ثابت کنید اگر تعداد سیاره ها فرد باشد سیاره ای وجود دارد که کسی آن را مشاهده نمی کند.

m_honarmand_j
14-12-2006, 15:53
سلام
در هر یک از خانه های یک جدول n*n که در آن n فرد است یکی از اعداد 1 یا 1- را به دلخواه نوشته ایم . زیر هر ستون حاصلضرب همه ی عدد های این ستون و در سمت راست هر سطر حاصلضرب همه ی عدد های آن سطر را نوشته ایم . ثابت کنید مجموع همه ی این 2n حاصلضرب نمی تواند برابر صفر شود .

m_honarmand_j
14-12-2006, 17:16
سلام
در هر خانه از جدول n*m حرفی قرار داده شده است. می دانیم همه ی سطر های جدول با هم فرق دارند . ثابت کنید ستونی وجود دارد که بعد از پاک کردن آن در جدول باقی مانده باز هم سطر های یکسان پیدا نمی شود .

m_honarmand_j
14-12-2006, 17:19
سلام
در یک جدول n*n ، به دلخواه n-1 خانه را سیاه می کنیم . (سایر خانه ها سفیداند) . در هر مرحله خانه ای که دو یا بیشتر همسایه ی سیاه دارد را سیاه می کنیم . آیا با این کار می توان همه ی خانه ها را رنگ کرد؟

m_honarmand_j
14-12-2006, 17:23
سلام
خط های هوایی بین شهر های یک کشور طوری تنظیم شده اند که هر شهر حد اکثر با سه شهر ارتباط دارد . در ضمن برای مسافرت از هر شهر به هر شهر دیگر حداکثر یک بار تعویض هواپیما نیاز است . حد اکثر تعداد شهر های این کشور چند تا است . (برای این شهر ها مسیر های هوایی را مشخص کنید.)

m_honarmand_j
14-12-2006, 17:32
سلام
2000 سکه وجود دارد و دو نفر می خواهند بازی ای انجام دهند . نفر اول سکه ها را به دو دسته که حداقل دو سکه دارند تقسیم می کند . نفر دوم هر دسته را به دو دسته ی ناتهی تقسیم کرده و دسته هایی که بیشترین و کمترین مقدار را دارا هستند بر می دارد . نفر اول حد اکثر از بردن چند سکه می تواند مطمئن شود؟

m_honarmand_j
16-12-2006, 15:08
سلام بر همه ی دوستان
می خواستم یک توضیحی بدم . من خودم بعضی از همین سؤالات را حل نکردم و راه حل شون و نمی دونم . ولی اکثر شون و حل کردم و حل اونا رو پس از مدتی براتون می ذارم . به همین دلیل اگه سؤالی رو سعی می کنید حل کنید ولی نتونستید ، تا هر جا که رفتید تو اینجا مطرح کنید چون ممکنه ایده ای که به کار می برید جالب با شه یا حتی مقداری از مسئله رو حل کرده باشید و در یک قسمتش مشکل داشته باشید که دیگران بتونن کار شما رو تکمیل کنن .
پس هیچ وقت ناامید نشید .
باتشکر از همه

m_honarmand_j
16-12-2006, 15:09
سلام
در یک تورنونمت 10 نفر شرکت کرده اند . اگر x1 بردهای نفر اول ، x2 بردهای نفر دوم و ... و y1 باخت های نفر اول ، y2 باختهای نفر دوم و ... باشد . ثابت کنید :
x1+x2+…+x10=y1+y2+…+y10

m_honarmand_j
16-12-2006, 15:10
سلام
n عدد داریم . دونفر بازی زیر را انجام می دهند . هر کس در نوبت خود عددی را انتخواب و درمجموعه ی A قرار می دهد . اگر عدد انتخاب شده مقسوم علیه یکی از اعداد مجموعه ی A باشد آن فرد می بازد . استراتژی برد را بیابید .

m_honarmand_j
16-12-2006, 15:18
سلام
یک مکعب داریم . دونفر بازی زیر را روی این مکعب انجام می دهند . هر کس در نوبت خود سه تا از ضلع های این مکعب را رنگ می کند . کسی که ضلعهای یک وجه را به رنگ خودش در آورد برنده می شود . آیا نفر اول می تواند طوری بازی کند که همیشه برنده شود؟

m_honarmand_j
17-12-2006, 15:24
سلام
روی هر سیاره از یک منظومه یک اختر شناس وجود دارد و هر اختر شناس نزدیکترین سیاره را مشاهده می کند . فاصله ی بین سیاره ها دو به دو متمایز است . ثابت کنید اگر تعداد سیاره ها فرد باشد سیاره ای وجود دارد که کسی آن را مشاهده نمی کند.

سلام
برای حل این سؤال تقریبا می شه گفت که استقرا یی کار می کنیم . یا بخواهیم بهتر بگیم یه چیزایی و سعی می کنیم حذف کنیم . دو سیاره که کمترین فاصله رو با هم دارند در نظر می گیریم . دو اختر شناسی که روی این دو سیاره هستند مجبورند همدیگر رو ببینند . پس می تونیم این دو نفر و این دو سیاره رو در نظر نگیریم و آنها رو کنار بزاریم . اما ممکنه که سیاره ای باشه که کمترین فاصله رو از یکی از این دوسیاره داشته باشه . پس اون فرد که روی این سیاره است و می زاریم کنار ولی سیاره رو نگه می داریم . به همین ترتیب پیش می ریم . چون تعداد سیارات فرد است در آخر سیاره ای تنها می ماند در نتیجه کسی آن را نمی بیند . :biggrin:

ali_hp
17-12-2006, 22:32
سلام
در یک جدول n*n ، به دلخواه n-1 خانه را سیاه می کنیم . (سایر خانه ها سفیداند) . در هر مرحله خانه ای که دو یا بیشتر همسایه ی سیاه دارد را سیاه می کنیم . آیا با این کار می توان همه ی خانه ها را رنگ کرد؟

سلام
نمی توان همه خانه ها را رنگ کرد.
به سادگی می توان دید که عمل تعریف شده محیط شکل سیاه رنگ را زیادنمی کند
ودر حالت اولیه محیط شکل سیاه رنگ حداکثر 4n-4 است
(در حالتی که هیچ دو تایی از این n-1 مربع ضلع مشترک نداشته باشند محیط 4n-4 است)
اما اگر همه صفحه سیاه شود باید محیط شکل سیاه رنگ 4n بشود.

ali_hp
17-12-2006, 22:42
سلام
در یک تورنونمت 10 نفر شرکت کرده اند . اگر x1 بردهای نفر اول ، x2 بردهای نفر دوم و ... و y1 باخت های نفر اول ، y2 باختهای نفر دوم و ... باشد . ثابت کنید :
x1+x2+…+x10=y1+y2+…+y10
سلام
هر یک از طرفین تساوی بالابرابر است با:
{تعداد کل بازیها} - {تعداد تساوی ها}

ali_hp
17-12-2006, 23:44
سلام
در هر یک از خانه های یک جدول n*n که در آن n فرد است یکی از اعداد 1 یا 1- را به دلخواه نوشته ایم . زیر هر ستون حاصلضرب همه ی عدد های این ستون و در سمت راست هر سطر حاصلضرب همه ی عدد های آن سطر را نوشته ایم . ثابت کنید مجموع همه ی این 2n حاصلضرب نمی تواند برابر صفر شود .
سلام
p(k)l برابر حاصلضرب اعداد سطر kام وq(k)l برابر حاصلضرب اعدادستون k ام است.
حاصلضرب این 2n عدد=p(1)p(2)...p(n)q(1)q(2)...q(n)l={حاصلض رب اعداد جدول} به توان دو=یک
اگر جمع این 2n عدد برابر صفر شودباید n تای انها یک و n تای انها منفی یک باشد.پس حاصلضرب این 2n عدد می شود {منفی یک}به توان{n} که اگر n فرد باشد برابر منفی یک می شود که غیر ممکن است.

m_honarmand_j
19-12-2006, 23:11
سلام
هر یک از طرفین تساوی بالابرابر است با:
{تعداد کل بازیها} - {تعداد تساوی ها}

سلام
ببخشید من تو این سؤال یه سوتی دادم . سوتیم اینه که تو مسوله همه ی x ها و همه ی y ها توان دو داشتن و من موقع نوشتن اونارو فراموش کردم . باز هم پوذش می خوام .

m_honarmand_j
19-12-2006, 23:17
سلام
p(k)l برابر حاصلضرب اعداد سطر kام وq(k)l برابر حاصلضرب اعدادستون k ام است.
حاصلضرب این 2n عدد=p(1)p(2)...p(n)q(1)q(2)...q(n)l={حاصلض رب اعداد جدول} به توان دو=یک
اگر جمع این 2n عدد برابر صفر شودباید n تای انها یک و n تای انها منفی یک باشد.پس حاصلضرب این 2n عدد می شود {منفی یک}به توان{n} که اگر n فرد باشد برابر منفی یک می شود که غیر ممکن است.

سلام
راه حل دروسته و منم راه حل دیگه ای برای این مسئله پیدا نکردم .

m_honarmand_j
19-12-2006, 23:24
سلام
نمی توان همه خانه ها را رنگ کرد.
به سادگی می توان دید که عمل تعریف شده محیط شکل سیاه رنگ را زیادنمی کند
ودر حالت اولیه محیط شکل سیاه رنگ حداکثر 4n-4 است
(در حالتی که هیچ دو تایی از این n-1 مربع ضلع مشترک نداشته باشند محیط 4n-4 است)
اما اگر همه صفحه سیاه شود باید محیط شکل سیاه رنگ 4n بشود.

سلام علی جان
راه حلت درسته و منم از همین راه حلش کرده بودم و این یکی از معدود مسائله که ایده اش استفاده از محیطه .
موفق باشی

m_honarmand_j
20-12-2006, 16:21
سلام
در هر خانه از جدول n*m حرفی قرار داده شده است. می دانیم همه ی سطر های جدول با هم فرق دارند . ثابت کنید ستونی وجود دارد که بعد از پاک کردن آن در جدول باقی مانده باز هم سطر های یکسان پیدا نمی شود .

سلام
به ازای هر سطر رأسی را در نظر می گیریم . فرض کنید پس از پاک کردن ستون اول سطر اول با سطر دوم یکی شوند . پس این دو رأس را به هم وصل می کنیم . پس از پاک کردن ستون دوم سطر دوم با سطر سوم و ... و پس از پاک کردن ستون n ام سطر m با سطر یکم یکی شوند پس آنها را به هم وصل می کنیم . (به هر صورتی که پیش برویم) به سادگی مشاهده می شود که یک دور در این گراف بوجود می آید . این یعنی اینکه سطر ...

ادامه ی اثبات رو بر عهده ی شما می زارم . ببینید می تونید با این راهنمایی این مسئله رو حل کنید .

m_honarmand_j
20-12-2006, 16:30
سلام
خط های هوایی بین شهر های یک کشور طوری تنظیم شده اند که هر شهر حد اکثر با سه شهر ارتباط دارد . در ضمن برای مسافرت از هر شهر به هر شهر دیگر حداکثر یک بار تعویض هواپیما نیاز است . حد اکثر تعداد شهر های این کشور چند تا است . (برای این شهر ها مسیر های هوایی را مشخص کنید.)

سلام
به سادگی می شه دید که تعداد شهر ها 10 تا است و فقط باید راه هارو پیدا کنید .
جواب مسئله گراف پترسن است . :tongue:

m_honarmand_j
20-12-2006, 16:33
سلام
2000 سکه وجود دارد و دو نفر می خواهند بازی ای انجام دهند . نفر اول سکه ها را به دو دسته که حداقل دو سکه دارند تقسیم می کند . نفر دوم هر دسته را به دو دسته ی ناتهی تقسیم کرده و دسته هایی که بیشترین و کمترین مقدار را دارا هستند بر می دارد . نفر اول حد اکثر از بردن چند سکه می تواند مطمئن شود؟

سلام
نفر اول سکه ها رو به دو دسته ی مساوی تقسیم می کند . نفر دوم هر کاری بکند به نفر اول نصف سکه ها می رسد . می شود تحقیق کرد که هیچ گاه بیشتر از نصف سکه ها به نفر اول نمی رسد . :happy:

m_honarmand_j
20-12-2006, 16:37
سلام
مجموعه ی n عضوی چند زیر مجموعه ی غیر متقاطع دارد ؟

m_honarmand_j
20-12-2006, 16:38
سلام
خانه های صفحه ی شطرنج n*n که در آن n زوج و برگتر از 2 می باشد را با n*n)/2) رنگ مختلف به نحوی رنگ می کنیم که هر رنگ درست برای دو خانه به کار رفته است . ثابت کنید می توان n رخ را طوری قرار داد که در خانه هایی با رنگ های مختلف قرارر داشته باشند و در ضمن یکدیگر را تحدید نکنند.

m_honarmand_j
20-12-2006, 16:53
سلام
در یک تورنونمت 10 نفر شرکت کرده اند . اگر x1 بردهای نفر اول ، x2 بردهای نفر دوم و ... و y1 باخت های نفر اول ، y2 باختهای نفر دوم و ... باشد . ثابت کنید :
x1+x2+…+x10=y1+y2+…+y10

سلام
اولا بگم که قبلا گفتم توی اسن یؤال یه سوتی داده بودم و همه ی x ها و y ها توان دو دارند .
همه ی y ها رو به طرف دیگر تساوی منتقل می کنیم . پس داریم : x1 به توان دو منهای y1 به توان دو و ...
از اتحاد استفاده کرده و تبدیل می شود به (x1+y1)(x1-y1) و ... که مجموع برد ها و باختهای هر نفر برابر تعداد بازی ها مساوی 9 است . پس می توان از 9 فاکتور گرفت و مسئله تبدیل می شود به همونی که اول به اشتباه گفته بودم.

m_honarmand_j
20-12-2006, 17:44
سلام
n عدد داریم . دونفر بازی زیر را انجام می دهند . هر کس در نوبت خود عددی را انتخواب و درمجموعه ی A قرار می دهد . اگر عدد انتخاب شده مقسوم علیه یکی از اعداد مجموعه ی A باشد آن فرد می بازد . استراتژی برد را بیابید .

سلام
اگر نفر اول استراتژی برد داشته باشد آن را انجام می دهد .حال فرض کنید بازیکن نفر اول استراتژی برد ندارد .
نفر اول عدد یک را انتخاب می کند(چون اگر هر عددی را انتخاب کنیم عدد یک حذف می شود) و نوبت نفر دوم می شود و انگار او آغاز کننده ی بازی است پس می بازد . پس همیشه نفر اول برنده است .

m_honarmand_j
20-12-2006, 17:50
سلام
یک مکعب داریم . دونفر بازی زیر را روی این مکعب انجام می دهند . هر کس در نوبت خود سه تا از ضلع های این مکعب را رنگ می کند . کسی که ضلعهای یک وجه را به رنگ خودش در آورد برنده می شود . آیا نفر اول می تواند طوری بازی کند که همیشه برنده شود؟

سلام
می توان دید که نفر اول هر طوری انتخاب های خود را انجام دهد باز هم نفر دوم می تواند سه ضلع را طوری انتخاب کند تا همه ی شش وجه را پوشش دهد .

m_honarmand_j
23-12-2006, 19:34
سلام
40 ضلعی منتظمی مفروض است . آیا می توان رأس های آن را با اعداد 0 تا 9 طوری شماره گزاری کرد که برای هر دو عدد مختلف ضلعی وجود داشته باشد که انتهای آن این اعداد شماره گزاری شده باشد؟

m_honarmand_j
23-12-2006, 19:35
سلام
جهانگردی با قطار به مسکو آمده بود . تمام روز را در شهر قدم زد . در رستورانی واقع در یک میدان شام خورد و تصمیم گرفت به استگاه برگردد و در ضمن تنها از خیاباهایی عبور کند که تا آن لحضه به تعداد فرد از آنها عبور کرده است . ثابت کنید جهانگرد در هر حال می تواند به این ترتیب خود را به استگاه برساند .

m_honarmand_j
23-12-2006, 19:36
سلام
n نقطه داده شده است و n<4 . ثابت کنید می توان این نقطه ها را با پیکان چنان به هم وصل کرد که از هر نقطه با حداکثر به کمک دو پیکان به هر نقطه ی دیگر رفت .

m_honarmand_j
23-12-2006, 19:37
سلام
5 نفر هستند به طوری که از هر 3 نفر 2 نفر همدیگر را می شناسند . ثابت کنید که می توان آن ها را طوری به دور میزی چید که هر نفر افراد کنار خود را بشناسد .

m_honarmand_j
23-12-2006, 19:46
سلام بر دوستان
فرا رسیدن ایام امتحانات رو به بعضی ها تبریک و به بعضی ها تصلیت عرض می کنم.

m_honarmand_j
24-12-2006, 23:43
سلام بر دوستان
مدتی هست که من منبع مناسبی برای سوال حل کردن ندارم . اگه کسی کتابی ، سایتی یا هر منبع دیگه ای سراغ داره لطفا بهم معرفی کنه .
ممنون می شم .

m_honarmand_j
25-12-2006, 00:08
سلام
مجموعه ی n عضوی چند زیر مجموعه ی غیر متقاطع دارد ؟

سلام
به ازای هر عضو از مجموعه ببینید چند حالت وجود دارد . یا در یک زیر مجموعه هست و یا نیست پس ...:biggrin:

m_honarmand_j
25-12-2006, 00:35
سلام
40 ضلعی منتظمی مفروض است . آیا می توان رأس های آن را با اعداد 0 تا 9 طوری شماره گزاری کرد که برای هر دو عدد مختلف ضلعی وجود داشته باشد که انتهای آن این اعداد شماره گزاری شده باشد؟

سلام
به سادگی می توان دید که نمی شود .
این که باید به ازای هر دو عدد یک ضلع وجود داشته باشد و اعداد ما از 0 تا 9 است (10 تا) یعنی انتخاب 2 از 10 که مساوی است با 45 و ما 40 تا ضلع داریم .

m_honarmand_j
25-12-2006, 00:49
سلام
خانه های صفحه ی شطرنج n*n که در آن n زوج و برگتر از 2 می باشد را با n*n)/2) رنگ مختلف به نحوی رنگ می کنیم که هر رنگ درست برای دو خانه به کار رفته است . ثابت کنید می توان n رخ را طوری قرار داد که در خانه هایی با رنگ های مختلف قرارر داشته باشند و در ضمن یکدیگر را تحدید نکنند.


سلام
فرض کنید که دو رنگ هستند که در هر کدام از آنها یک رخ وجود دارد و این رخ ها همدیگر را تحدید می کنند (در یک سطر یا یک ستون اند) ببینید چند حالت رخ می دهد . سپس تهداد کل حالت هایی که می توان رخ ها را در صفحه چید را بیابید . ببینید به چه نتیجه ای می رسید . ( در واقع می بینیم که تعداد حالت های نامطلوب از کل حالت ها کمتر است در نتیجه حالتی وجود دارد که مطلوب است .) :tongue:

ali_hp
25-12-2006, 01:10
سلام بر دوستان
مدتی هست که من منبع مناسبی برای سوال حل کردن ندارم . اگه کسی کتابی ، سایتی یا هر منبع دیگه ای سراغ داره لطفا بهم معرفی کنه .
ممنون می شم .
سلام
[ برای مشاهده لینک ، لطفا با نام کاربری خود وارد شوید یا ثبت نام کنید ] سایته خیلی خوبیه.اگه قبلا ندیدیش حتما یه سر بزن.

ali_hp
25-12-2006, 01:27
سلام
اگر نفر اول استراتژی برد داشته باشد آن را انجام می دهد .حال فرض کنید بازیکن نفر اول استراتژی برد ندارد .
نفر اول عدد یک را انتخاب می کند(چون اگر هر عددی را انتخاب کنیم عدد یک حذف می شود) و نوبت نفر دوم می شود و انگار او آغاز کننده ی بازی است پس می بازد . پس همیشه نفر اول برنده است .
سلام
من این راه حل نفهمیدم خوب اگه عدد یک نباشه چی؟
اگر اعداد اولیه طوری باشند که هیچیک بر دیگری بخش پذیر نباشد نفر اول نمی تواند بازی را ببرد( بازی برنده ندارد)

m_honarmand_j
28-12-2006, 16:21
سلام
من این راه حل نفهمیدم خوب اگه عدد یک نباشه چی؟
اگر اعداد اولیه طوری باشند که هیچیک بر دیگری بخش پذیر نباشد نفر اول نمی تواند بازی را ببرد( بازی برنده ندارد)

سلام بر دوست عزیزم
این بار اگه بزنی در گوشم هم حق داری . این n تا عدد از 1 تا n هستند .

m_honarmand_j
28-12-2006, 16:34
سلام
n نقطه داده شده است و n<4 . ثابت کنید می توان این نقطه ها را با پیکان چنان به هم وصل کرد که از هر نقطه با حداکثر به کمک دو پیکان به هر نقطه ی دیگر رفت .

سلام بر دوستان
فقط کافی است که برای n به ازای 5 و 6 این پیکان ها را پیدا کنیم و سپس اگر n فرد باشد از 5 و اگر زوج باشد از 6 استفاده کنیم به این ترتیب که رأس های قبلی را در یک خط می چینیم و دو رأی جدید را یکی در بالا و یکی را در پایین می گذاریم و سپس همه ی یالهایی که با رأس بالا وصل هستند به جز یالی که شامل رأس جدید است را به سمت خارج از رأس جدید پیکان می کشیم و همه ی یالهای رأس جدید دیگر را به سمت آن رأس پیکان می کشیم . در آخر یالی که بین دو رأس جدید است را به طرف رأس بالایی پیکان می کشیم .
امیدوارم که فهمیده باشید که چی گفتم چون خودم نفهمیدم. :biggrin:

m_honarmand_j
28-12-2006, 16:41
سلام
5 نفر هستند به طوری که از هر 3 نفر 2 نفر همدیگر را می شناسند . ثابت کنید که می توان آن ها را طوری به دور میزی چید که هر نفر افراد کنار خود را بشناسد .

سلام
باید ثابت کرد که هر نفر دقیقا دونفر دیگر را می شناسد . در گرافی که درجه ی هر رأس 2 باشد حتما دوری وجود دارد که شامل همه ی آنها است ( البته باید توجه داشت که گراف همبند باشد) . پس افراد را طبق آن دور کنار هم می چینیم . :tongue:

m_honarmand_j
28-12-2006, 17:23
سلام بر دوستان
یه سؤال جالب .
n رأس و 100 یال موجود است . در هر مرحله می توانیم رأسی را انتخاب کنیم و تمام یال های متصل به آن را حذف و آن را به تمام رأس هایی که قبلا به آن ها وصل نبوده وصل کنیم . بعد از انجام چند مرحله می توان مطمئن بود که بتوان دو رأس را یافت به طوری که هیچ مسیری بین آن ها نباشد .

m_honarmand_j
28-12-2006, 17:24
سلام
آدمی را گوشه گیر می نامیم که کمتر از 10 آشنا داشته باشند و آدمی را عجیب می نامیم که همه ی آشنایان او افراد گوشه گیر باشند (آشنایی رابطه ای متقارن است). اگر m آدم گوشه گیر و n آدم عجیب وجود داشته باشند ثابت کنید که m بیشتر یا مساوی با n است .

m_honarmand_j
28-12-2006, 17:25
سلام
یک سازنده ی اسباب بازی در هر روز حداقل یک اسباب بازی درست می کند ولی در طول سال توان ساخت بیش از 725 اسباب بازی را ندارد . به ازای هر عدد طبیعی مانند n ثابت کنید چند روز متوالی وجود دارد که این سازنده طی آنها دقیقا n اسباب بازی ساخته است (سال را 365 روز در نظر بگیرید).

m_honarmand_j
28-12-2006, 17:52
سلام بر دوستان
خیلی ممنون که این همه در بحث شرکت می کنیم . بابا یه نظری یه چیزی یه ...
باور کنید اگه بهم فش هم بدید حد اقل می گم دونفر می آن سر می زنن و خوشحال می شم .
بابا چند تا هم شما ها سؤال بزارید .
ممنون می شم .

m_honarmand_j
29-12-2006, 15:17
سلام
18 رأس داده شده اند به طوری که هیچ سه تایی از آنها روی یک خط نیستند . در نتیجه آنها 816 مثلث تشکیل می دهند . مجموع فضاهای این مثلثها را با A نمایش می دهیم . شش تا از آنها قرمز ، شش تا سبز و شش تا آبی هستند . نشان دهید مجموع فضاهایی که مربوط به مثلث هایی رأس هایش یک رنگ اند از A/4 متجاوز نمی شود .

eh_mn
31-12-2006, 23:27
سلام
یک سازنده ی اسباب بازی در هر روز حداقل یک اسباب بازی درست می کند ولی در طول سال توان ساخت بیش از 725 اسباب بازی را ندارد . به ازای هر عدد طبیعی مانند n ثابت کنید چند روز متوالی وجود دارد که این سازنده طی آنها دقیقا n اسباب بازی ساخته است (سال را 365 روز در نظر بگیرید).
سلام.

براي هر n ؟؟

m_honarmand_j
01-01-2007, 19:13
سلام eh_mn
n بین 1 و 725 است . (البته تو متن سؤال نیست و لی حتمی باید بین 1 و 725 باشه )

eh_mn
02-01-2007, 16:56
سلام eh_mn
n بین 1 و 725 است . (البته تو متن سؤال نیست و لی حتمی باید بین 1 و 725 باشه )
فرض كن n=725 . در اين صورت به نظر شما آيا مي توان حداقل دو روز پيدا كرد كه در هر دو روز n تا اسباب بازي ساخته شده باشد ؟

m_honarmand_j
03-01-2007, 23:17
سلام بر دوستان
ببخشید که چند روزه سؤالی نذاشتم چون درگیر امتحانات هستم . حالا بماند که معلما هم دارند طوری امتحان می گیرن که انگار ارث باباشونو خوردیم .
در ضمن در جواب eh_mn هم باید بگم داداش ، اگه n=725 باشه تعداد روز هارو کل سال می گیریم .
سؤال نگفته که حتمی در فلان روز متوالی . فقط باید ثابت کرد که در چند روز متوالی که این چند روز بین 1 و 365 باشه .
اگه بازم مشکلی بود دوباره مطرح کنید لطفا .

m_honarmand_j
03-01-2007, 23:34
سلام
گرافی 1991 رأس دارد که مینیمم درجه ی آن 1593 است . ثابت کنید که شش رأس وجود دارند که همگی به هم وصل هستند . آیا 1593 بهترین امکان است ؟

m_honarmand_j
03-01-2007, 23:49
سلام
997 نقطه در صفحه داده شده اند . نشان دهید که حداقل 1991 نقطه ی میانی جدا از هم وجود دارد . آیا امکان پذیر است که دقیقا 1991 نقطه ی میانی (midpoint) داشته باشیم ؟

m_honarmand_j
03-01-2007, 23:54
سلام
یک گراف همبند با 1998 رأس داده شده که درجه ی هر رأس برابر 3 است . اگر 200 رأس که هیچ دوتایی از آنها متصل به هم با یک یال نیستند را پاک کنیم ، نشان دهید حاصل باز هم یک گراف همبند است .

ali_hp
04-01-2007, 01:08
سلام بر دوستان
در ضمن در جواب eh_mn هم باید بگم داداش ، اگه n=725 باشه تعداد روز هارو کل سال می گیریم .
سؤال نگفته که حتمی در فلان روز متوالی . فقط باید ثابت کرد که در چند روز متوالی که این چند روز بین 1 و 365 باشه .
اگه بازم مشکلی بود دوباره مطرح کنید لطفا .
سلام
مثلا فرض کنید کارخانه در هر روز دقیقا یک اسباب بازی تولید می کند در این صورت نمی توان چند روز متوالی بین یک تا 365 پیدا کرد که کارخانه در ان 725(یا هر عدد بیشتر از 365) تا اسباب بازی تولید کرده باشد.
منظور سوال این است که برای هر n در طول کل عمر نامتناهی کارخانه چند روز متوالی پیدا می شود(تعداد این روزها می تواند بیش از 365 باشد) که در مجموع n اسباب بازی تولید شده باشد.و مساله برای هر n درست است.(ممکن است این چند روز متوالی مثلا بعد از 5 سال اتفاق بیافتد)

m_honarmand_j
04-01-2007, 16:21
سلام
در جواب ali_hp باید بگم که احتمالا ایشون درست می فرمایند و این چند روز ممکن است در یک سال نباشد .
ولی شماها چه گیری به این دادین . نکته مهم اینه که در سال بیش از 725 نمی تواند درست کند . اما باید در صورت سوال می اومد که این فرد حتمی همه ی اون تعداد و باید در سال بسازه .
شما ها اول این مسئله رو حل کنید بعد برید سراغ اون مسئله
ورزش کاری 4 هفته برای آماده شدن در مسابقات فرصت دارد . فرض کنید می خواهد 40 جلسه تمرین کند . ثابت کنید که هر طور این 40 جلسه را در 28 روز کتوالی توزیع کند چند روز متوالی هست که در آن روزها درست 15 بار تمرین کرده باشد . با شرطی که هر روز حداقل 1 جلسه تمرین کند .

m_honarmand_j
04-01-2007, 16:28
سلام
یک جدول 9*9 داریم که پر از مورچه است که می توانند به خانه های قطری مجاور خود بروند . در یک حرکت تمام مورچه ها جا به جا می شوند . کمترین تعداد خانه های خالی چند تا است ؟

m_honarmand_j
04-01-2007, 18:16
سلام
در یک جدول 8*8 از گوشه ی سمت چپ پایین شروع به حرکت می کنیم و فقط مجاز به حرکت هایی به صورت (1،0) و (0،1) و (1،1) هستیم . آیا می توانیم از تمام خانه ها به طوری که از هر خانه فقط یک بار عبور کنیم ، بگذریم ؟

ali_hp
04-01-2007, 18:21
سلام
در جواب ali_hp باید بگم که احتمالا ایشون درست می فرمایند و این چند روز ممکن است در یک سال نباشد .
ولی شماها چه گیری به این دادین . نکته مهم اینه که در سال بیش از 725 نمی تواند درست کند . اما باید در صورت سوال می اومد که این فرد حتمی همه ی اون تعداد و باید در سال بسازه .
شما ها اول این مسئله رو حل کنید بعد برید سراغ اون مسئله
ورزش کاری 4 هفته برای آماده شدن در مسابقات فرصت دارد . فرض کنید می خواهد 40 جلسه تمرین کند . ثابت کنید که هر طور این 40 جلسه را در 28 روز کتوالی توزیع کند چند روز متوالی هست که در آن روزها درست 15 بار تمرین کرده باشد . با شرطی که هر روز حداقل 1 جلسه تمرین کند .
سلام
لازم نیست درصورت سوال بیادکه فردحتمی همه اون تعدادوباید در سال بسازه.
شاید اگه راه حلمو بگم بهتر باشه:
فرض کنید ak تعداد اسباب بازیهای تولید شده تا روز k ام باشد یعنی ak برابر است با:
تعداد تولید در روز اول+تعداد تولید در روز دوم+....+تعداد تولید در روز k ام.
چون در هر روز حداقل یک تولید داریم پس:a1<a2<...<ak از طرفی چون در هر سال حداکثر 725 تا تولید داریم نتیجه می شود :a(365m)=<725m (یعنی تولید تا پایان سال m ام حد اکثر 725m است) اگر چند روز متوالی وجود نداشته باشد که در انها مجموعا n اسباب بازی تولید شده باشد انگاه برای هر m همه اعداد زیر متمایزند:
a(1),a(2),...a(365m),a(1)+n,a(2)+n,...a(365m)+n ازطرفی تعداد این اعداد 730m است.و بزرگترین انها عدد
a(365m)+n است که از 725m+nکوچکتر مساوی است.پس باید 730m کوچکتر از 725m+n باشد که برای m بزرگتر از
n/5 غیر ممکن است.
در حقیقت ثابت کردیم اگر m>n/5 انگاه تا قبل از پایان سال m ام می توان چند روز متوالی یافت که در مجموع در ان روزها n تا اسباب بازی تولید شده باشد.

m_honarmand_j
04-01-2007, 18:28
سلام ali_hp جان
راه حلت جالب بود . اما اگه از اصل لانه کبوتری بری راحت تر به جواب می رسی .

ali_hp
04-01-2007, 18:42
سلام ali_hp جان
راه حلت جالب بود . اما اگه از اصل لانه کبوتری بری راحت تر به جواب می رسی .
بله کوتاه تر می شد.راه حل منم با اصل لانه کبوتری بود ولی نمی دونم چرا وقتی نوشتمش اینجوری شد:blink: .
این راه حلم در اصل همون اصل لانه کبوتری که روند اثبات اون اصلم تو خودش اورده.

m_honarmand_j
04-01-2007, 18:53
سلام
منم فهمیدم که با اصل لانه لبوتری رفتی ولی وستا ولش کردی .

m_honarmand_j
11-01-2007, 00:42
سلام
ثابت کنید که از بین هر شش نفر ، می توان سه نفر طوری جدا کرد که یا دو به دو با هم آشنا ، و یا دو به دو با هم نا آشنا باشند .

m_honarmand_j
11-01-2007, 00:46
سلام
2n+1 چیز در اختیار داریم . ثابت کنید که راه های انتخاب کردن دسته هایی با تعداد فرد برابر راه های انتخاب کردن دسته هایی با تعداد زوج است .

m_honarmand_j
11-01-2007, 00:48
سلام
درون یک مربع به ضلع واحد ، 51 نقطه داده شده است . ثابت کنید درون آنها سه نقطه وجود دارند که در داخل دایره ای به شعاع 7/1 جا بگیرند .

m_honarmand_j
11-01-2007, 02:15
سلام
در یک جدول 8*8 از گوشه ی سمت چپ پایین شروع به حرکت می کنیم و فقط مجاز به حرکت هایی به صورت (1،0) و (0،1) و (1،1) هستیم . آیا می توانیم از تمام خانه ها به طوری که از هر خانه فقط یک بار عبور کنیم ، بگذریم ؟

سلام
من این سؤال و با رنگ آمیزی حل کردم ولی حل دیگری هم براش دیدم .
صفحه رو شطرنجی می کنیم . حر کت هایی به صورت (0،1) و (1،0) رنگ خانه را تغییر می دهند . ولی حرکت (1،1) رنگ خانه را تغییر نمی دهد . اگر دنباله ای از خانه ها به صورت سفید ... سفید وجود داشته باشد باید دنباله ای دیگر به صورت سیاه...سیاه وجود داشته باشد که طول دو دنباله مساوی باشد . (چون تعداد خانه های سیاه و سفید برابر است) در نتیجه تعداد حرکت های (1،1) باید زوج باشد . واضح است که باید تعداد حرکت های (1،0) با حرکت های (0،1) برابر باشد و در نتیجه مجموع تعداد آنها زوج است . ولی ما باید تعداد فردی (63)حرکت انجام دهیم . در نتیجه این کار امکان پذیر نمی باشد . :rolleye:

m_honarmand_j
11-01-2007, 02:25
سلام
یک جدول 9*9 داریم که پر از مورچه است که می توانند به خانه های قطری مجاور خود بروند . در یک حرکت تمام مورچه ها جا به جا می شوند . کمترین تعداد خانه های خالی چند تا است ؟

سلام
جدول را به صورت شطرنجی رنگ می کنیم . در حرکت مورچه ها رنگ خانه ی آن ها عوض نمی شود . به راحتی می توان دید بهترین حالت وقتی رخ می دهد که دو مورچه جای خود را با هم عوض کنند . در نتیجه نه مورچه به خانه هایی باید بروند که در آنها مورچه وجود دارد . :sad:

ali_hp
11-01-2007, 02:29
سلام
من صورت سوال صفجه شطرنج 8*8 نفهمیدم.میشه بیشتر توضیح بدین؟
اگه اولین حرکت رو به با لا باشد واضح است که دیگر از خانه های ردیف پایین نمی توانیم بگذریم و اگر هم به سمت راست باشد دیگر از خانه های سمت چپ ترین ستون نمی توانیم بگذریم.اگه هم قطری باشد که نه از ردیف پایین می توان گذشت ونه از ردیف سمت چپ.

m_honarmand_j
11-01-2007, 03:07
سلام ali_hp
در اون سوال به جای حرکت (1،1) حرکت (1-،1-) رو بزار ببین جور می شه .

m_honarmand_j
11-01-2007, 03:20
سلام
n نفر به ضرب سکه مشغولند . برخی تنها سکه ی تقلبی و بقیه تنها سکه ی واقعی ضرب می کنند . وزن سکه ی تقلبی ، با وزن سکه ی واقعی فرق دارد . یک ترازو با گونه های مختلف وزنه و یک سکه ی واقعی در اختیار داریم . از هر کدام از ضرب کنندگان سکه ، هر قدر سکه که بخواهیم می توانیم بگیریم . چگونه می توان با سه بار وزن کردن همه ی کسانی را که به ساختن سکه ی تقلبی مشغولند ، شناخت؟

m_honarmand_j
11-01-2007, 18:14
سلام بر دوستان
امروز می خوام داستان به وجود آمدن استقرا رو در ریاضیات براتون بگم .
کشف استقرا مربوط به فردی ایتالیایی به نام Franciscus Maurolycus است .
در زمان های گذشته وقتی می گفتند x روز قبل از هالیدی ، مشکل اینجا بود که نمی دونستن که باید خود هالیدی و هم با اون روز ها حساب کنند یا نه .
استقرا از اینجا شروع شد که وقتی می گفتند یک روز قبل از هالیدی نمی توانست که منظورشون خود هالیدی هم باشه . در نتیجا یک روز قبل از هالیدی خود هالیدی و در بر نداشت . درتنیجه دو روز قبل از هالیدی هم نمی توانست که خود هالیدی و در بر داشته باشه چون در اون صورت دو روز قبل از هالیدی با یک روز قبل از هالیدی هم معنی می شد . همین طور سه روز قبل از هالیدی هم نمی توانست هالیدی را در بر داشته باشد و الی آخر .
این طور شد که استقرای ریاضی به وجود آمد .

m_honarmand_j
12-01-2007, 15:29
سلام بر دوستان
من یکم فکر کردم گفتم حالا که اتاق ترکیبیات زدم بیام بگم اصلا ترکیبیات چیه ؟
به همین خاطر این مطلب و براتون گذاشتم .
(منبع: جزوه ی دکتر سید عبداله محمودیان مدرس دانشکده ی علوم ریاضی دانشگاه صنعتی شریف)
آنالیز ترکیبی چیست؟
آنالیز ترکیبی که به آن ریاضیات ترکیبی و یا ترکیبیاتی نیز اطلاق می شود ، رشته ای از ریاضیات است که از دوران قدیم آغاز شده است . بنا به افسانه های نقل شده ، یو (yu) امپراطور چین (2200 سال قبل از میلاد) مربعی وفقی زیر را پشت لاک پشت "آسمانی" ملاحظه کرده بود .
2 9 4
7 5 3
6 1 8

جایگشت های آغازی از 1100 سال قبل از میلاد در چین هستند . بیشتر کارهای اولیه در ترکیبیات با اسرار اعداد پیوند خورده است ولی در طول چندین قرن اخیر ، نویسندگان مختلف از نظر یک سرگرمی ریاضی با این موضوع برخورد می کردند . مسأله ی وزنه های باچت (Bachet) ، مسئله ی دختر مدرسه ای های کرکمن (Kirkman) و مسئله ی 36 افسر اویلر از مثال های معروف هستند . مسائل فوق تفکر برانگیز بوده و گاهی اوقات حل آنها ابتکار آمیز و بسیار زیبا هستند .
بسیاری از مسائلی که در گذشته به صورت سرگرمی مطرح شده اند ، هم اکنون ارزش زیادی از نظر علوم محض و کاربردی یافته اند . چندی بیش نسیت که صفحه های تصویری متناهی به صورت یک مسئله کنجکاوانه ریاضیات ترکیبی مطرح می شد . ولی امروزه آنها اساس مبانی هندسه بوده و پایه ای بر تحلیل طرح های آزمایشی هستند . نیاز حیاتی تکنولوژی جدید عصر ما به مباحث کسسته ، از ریاضیات تفننی یک مبحث جدی و جدیدی به وجود آورده است . ولی مهمتر از همه اینکه عصر جدید برای ریاضیات ترکیبی یک حوزه وسیعی از مسائل جدید مجذوب کننده ایجاد کرده است . این مسائل در جبر مجرد ، توپولوژی ، مبانی ریاضی ، نظریه گرافها ، نظریه بازی ها ، برنامه ریزی خطی و بسیاری از موضوعات دیگر ریاضی به وجود آمده اند . ریاضیات ترکیبی دائما در حال دگرگونی بوده است . در زمان ما این دگرگونی از چندین جهت افزایش یافته است ، به طوری که با بسیاری از شاخه های مختلف ریاضی برخورد پیدا کرده است . در نتیجه دادن یک تعریف رسمی از آن مشکل است . ولی کلا می توان گفت که این علم مطالعه ای است روی آرایشها مختلف بر روی اعضای مجموعه ها . مجموعه ها معمولا متناهی بوده و آرایش ها هم با محدودیت هایی که مربوط به هر مسئله است همراه هستند . دو مسئله در حالت کلی مورد برسی هستند :
آیا یک پیکربندی به خصوص روی یک مجموعه ی متناهی وجود دارد یا خیر؟
در صورت وجود پیکر بندی به چند حالت است؟

soleares
12-01-2007, 15:36
دوست من خیلی متشکر کپی کردم ..

m_honarmand_j
12-01-2007, 15:59
خواهش می کنم عزیز .

m_honarmand_j
12-01-2007, 16:12
سلام
گرافی 1991 رأس دارد که مینیمم درجه ی آن 1593 است . ثابت کنید که شش رأس وجود دارند که همگی به هم وصل هستند . آیا 1593 بهترین امکان است ؟

سلام
رأسی را ماند v به دلخواه انتخاب کنید و فرض کنید که حداقل درجه یعنی 1593 است . سپس دیگر رأس ها را در دومجموعه قرار دهید . A مجموعه ای که با رأس مفروض همسایه اند و B رأس هایی که با رأس مفروض همسایه نیستند .
راسی را از همسایه های رأس مفروض در نظر گرفته و حداکثر همسایه های آن را در مجموعه ی B قرار دهید و ...
امید وارم بتونید بقیشو خودتون حل کنید .

m_honarmand_j
12-01-2007, 16:21
سلام
ثابت کنید که از بین هر شش نفر ، می توان سه نفر طوری جدا کرد که یا دو به دو با هم آشنا ، و یا دو به دو با هم نا آشنا باشند .

سلام
برای هر نفر یک رأس در نظر می گیریم .آشنا ها را با رنگ قرمز و نا آشنا ها را با رنگ سبز به هم وصل می کنیم . در این جمع رأسی به دلخواه را در نظر می گیریم . این فرد یا سه آشنا و یا سه نا آشنا در این جمع دارد . فرض می کنیم که سه آشنا دارد (قرمز). در بین این سه رأس اگر دو نفر همدیگر را بشناسند(با قرمز به هم وصل باشند) با نفر اول تشکیل یک مثلث (قرمز) می دهند و مسئله حل می شود و اگر هیچ کدام دیگری را نشناسد ، این سه نفر با هم تشکیل مثلثی (سبز) را میدهند و باز هم مسئله حل می شود . :laughing:

m_honarmand_j
12-01-2007, 16:24
سلام
2n+1 چیز در اختیار داریم . ثابت کنید که راه های انتخاب کردن دسته هایی با تعداد فرد برابر راه های انتخاب کردن دسته هایی با تعداد زوج است .

سلام
هر انتخاب مجموعه ای فرد از این اشیا موجب انتخاب نکردن مجموعه ای زوج می شود . :tongue:

m_honarmand_j
12-01-2007, 16:30
سلام
درون یک مربع به ضلع واحد ، 51 نقطه داده شده است . ثابت کنید درون آنها سه نقطه وجود دارند که در داخل دایره ای به شعاع 7/1 جا بگیرند .

سلام
هر ضلع مربع رو به پنج قسمت تقسیم می کنیم . در نتیجه مربع به 25 مربع کوچکتر تقسیم می شود . اگر در هر مربع کوچک دو نقطه باشد در مجموع می شود 50 نقطه در نتیجه در مربعی وجود دارد که در آن حداقل سه نقطه وجود دارد . دایره ای به شعاع 7/1 می تواند این مربع های کوچک را در خودش جادهد . در نتیجه مسئله حل می شود . ;)

ali_hp
12-01-2007, 18:59
سلام ali_hp
در اون سوال به جای حرکت (1،1) حرکت (1-،1-) رو بزار ببین جور می شه .
سلام
خوب اینجوری بهتره ولی جواب سوال هنوز غلطه!

eh_mn
16-01-2007, 06:58
سلام
2n+1 چیز در اختیار داریم . ثابت کنید که راه های انتخاب کردن دسته هایی با تعداد فرد برابر راه های انتخاب کردن دسته هایی با تعداد زوج است .
سلام.
من منظور اين سوال را متوجه نشدم. "دسته هايي با تعداد فرد" يعني دسته هايي كه تعداد اعضاي هر دسته فرد است يا اينكه تعداد دسته ها فرد است؟

m_honarmand_j
18-01-2007, 11:43
سلام بر دوستان
من خیلی خوشحال شدم که چند نفری هستن که نظر می دن و وقتی من مطلبی رو ناقص می نویسم بهم می گن . چون برای من مهمه که بتونم مطالبو انتقال بدم چون می خوان تو المپیاد شرکت کنم و مصحح های باشگاه خیلی سختگیرن . پس هرچی بیشتر شما بهم ایرادامو بگید من بیشتر پیشرفت می کنم .
خیلی ممنون .

m_honarmand_j
18-01-2007, 11:46
سلام
خوب اینجوری بهتره ولی جواب سوال هنوز غلطه!

سلام
من خودمم به شک اوفتادم . بزار دوباره فکر کنم جواب دقیق و برات می ذارم .

m_honarmand_j
18-01-2007, 11:52
سلام.
من منظور اين سوال را متوجه نشدم. "دسته هايي با تعداد فرد" يعني دسته هايي كه تعداد اعضاي هر دسته فرد است يا اينكه تعداد دسته ها فرد است؟

سلام
منظور تعداد اعضای هر دسته است . مثلا اگر مجموعه کل ما M باشد . زیر مجموعه ی A که تعداد اعضای آن را با نماد |A| نشان می دهند ،اگر فرد باشد ، متمم A را اگر مجموعه B بگیریم ، آنگاه |B| زوج خواهد بود .

m_honarmand_j
18-01-2007, 11:54
سلام
ثابت کنید در یک جمع 18 نفره یا چهار نفر هستند که همگی همدیگر را می شناسند یا چهاد نفر هستند که هیچکدام دیگری را نمی شناسد .

m_honarmand_j
18-01-2007, 12:02
سلام
روی یک صفحه ، m نقطه داده شده است . در ضمن همه ی آن ها روی یک خط راست نیستند . ثابت کنید دست کم می توان به تعداد (m-1)(m-2) (1/2) مثلث پیدا کرد .

m_honarmand_j
18-01-2007, 12:06
سلام
هشت رخ را در خانه های صفحه شطرنج قرار دادیم به طوری که هیچ دوتایی همدیگر را تهدید نمی کنند . ثابت کنید تعداد رخ های خانه های سیاه زوج است .

m_honarmand_j
28-01-2007, 19:50
سلام
ثابت کنید در یک جمع 18 نفره یا چهار نفر هستند که همگی همدیگر را می شناسند یا چهاد نفر هستند که هیچکدام دیگری را نمی شناسد .

سلام
هر فرد در بین دیگران یا دست کم 9 نفر آشنا و یا نا آشنا دارد . بدون اینکه به کلی بودن راه حل لطمه ای بخورد می توان فرض کرد ، فرد مشخص A ،در این جمع دست کم 9 نفر آشنا دارد . یکی از این 9 نفر ، و مثلا B ، را در نظر می گیریم . اگر بین 8 نفر دیگر 6 نفر پیدا شوند که هیچ کدام با B آشنا نباشند آنگاه مسئله منجر به مسئله ای می شود که در آن می گوییم بین هر 6 نفر 3 آشنای دوطرفه یا سه نا آشنای دوطرفه وجود دارد می شود . فرض می کنیم B ، در بین 8 نفر دیگر ، سه آشنا و پنج نا آشنا داشته باشد ( زیرا اگر B با چهار نفر آشنا باشد ، این چهار نفر یا دوبهدو با هم نا آشنا هستند که ، در این صورت ، یک گروه چهار نفری مورد نظر تشکیل می شود ، و یا دست کم دو نفر آن ها یکدیگر را می شناسند که ، دز این صورت ، A و B و این دونفر ، گروه مورد نظر را تشکیل می دهند.)
اکنون تعداد آشناها را در این 9 نفر محاسبه می کنیم : روی هم 9 نفرند، هر نفر درت با سه نفر آشناست ، چون هر آشنا دوبار به حساب می آید ، بنا بر این تعداد زوج آشنا ها برابر 2/(3*9) = 5/14 می شود که معنا ندارد . این تناقض ، حل مسئله را تمام می کند .

m_honarmand_j
28-01-2007, 19:53
سلام
اعداد فرد را به صورت زیر نوشته ایم .
1 3 5 7 9
11 13 15 17 19
21 23 25 27 29
31 33 35 37 39
41 43 45 47 49

از این عدد ها 5 عدد طوری انتخاب می کنیم که هیچ دوتایی در یک سطر یا ستون نباشند . ثابت کنید که حاصل جمع 5 عدد همیشه ثابت است .

m_honarmand_j
28-01-2007, 19:55
سلام
K رنگ در اختیار داریم . با چند روش می توان ضلع های یک n ضلعی منتظم را رنگ کرد به نحوی که ضلع های مجاور رنگ های مختلف داشته باشند ؟ (چند ضلعی را نمی توان چرخاند)

m_honarmand_j
28-01-2007, 19:55
سلام
صفحه کاغذ شطرنجی بی پایانی را با 9 رنگ مختلف رنگ کرده ایم . هر خانه یک رنگ دارد و در ضمن از همه ی رنگ ها استفاده شده است . دو رنگ را مجاور می نامیم هرگاه دوخانه که با یک ضلع مشترک پیدا شود که با این دو رنگ ، رنگ شده باشند . حداقل تعداد زوج رنگ های مجاور چند تاست؟

m_honarmand_j
28-01-2007, 19:56
سلام
روی یک صفحه ، m نقطه داده شده است . در ضمن همه ی آنها ، روی یک خط راست نیستند . ثابت کنید دست کم به تعداد 2/1 * (m-1) * (m-2) مثلت می توان رسم کرد که راس های آن روی این نقطه ها باشند .

m_honarmand_j
28-01-2007, 19:57
سلام
در مربع 5*5 ، 16 خانه را رنگ کرده ایم . ثابت کنید ، می توان در آن ، یک مربع 2*2 پیدا کرد که دست کم سه خانه ی آن ، رنگ شده باشد .

m_honarmand_j
28-01-2007, 19:58
سلام
یال های یک گراف کامل را که 2*n+1 راس دارد ، با سه رنگ مختلف رنگ کرده ایم. ثابت کنید ، می توان یکی از رنگ ها و n+1 راس را طوری انتخاب کرد که ، از هر یک از این راس ها به هر راس دیگر از آن ها ، بتوان ار طریق یال هایی حرکت کرد که دارای رنگ انتخابی ما باشند .

m_honarmand_j
28-01-2007, 20:03
سلام
راس های یک گراف محدود با دورنگ مختلف رنگ شده اند . در هر ثانیه ، هر نقطه تغییر رنگ می دهد و به رنگی در می آید که در همسایگی آن بیشتر است . ثابت کنید ، برای هر نقطه لحظه ای فرا می رسد که ، بعد از آن ، یا تغییر رنگ نمی دهد و یا در هر ثانیه ، تغییر رنگ می دهد .

m_honarmand_j
28-01-2007, 20:15
سلام
روی یک صفحه ، m نقطه داده شده است . در ضمن همه ی آن ها روی یک خط راست نیستند . ثابت کنید دست کم می توان به تعداد (m-1)(m-2) (1/2) مثلث پیدا کرد .

سلام
بدترین حالت زمانی است که m-1 نقطه روی یک خط باشند . برای تشکیل مثلث باید یک نقطه که روی خط نیست و دوتا از نقطه های روی خط را انتخاب کرد که انتخاب 2 از m-1 برابر همان چیزی است که خواسته بودیم .

m_honarmand_j
28-01-2007, 23:23
سلام
هشت رخ را در خانه های صفحه شطرنج قرار دادیم به طوری که هیچ دوتایی همدیگر را تهدید نمی کنند . ثابت کنید تعداد رخ های خانه های سیاه زوج است .

سلام
خانه ی شطرنج ، وقتی و تنها وقتی سفید است که مجموع شماره های ستون و سطر آن فرد باشد . چون مجموع شماره های سطر ها و ستون ها برای 8 رخ مفروض ، برابر است با 72=(8+...+2+1)2 که عددی زوج است ، بننابر این باید تعداد خانه های با شماره های فرد که در این مجموعه وجود دارند زوج باشد . یعنی تعداد خانه های سیاهی که شامل رخ هستند ، عددی است زوج .:blink:

m_honarmand_j
28-01-2007, 23:33
سلام
این بار یک مقاله آموزشی گذاشتم . این مقاله در مورد نظریه بازی ها است . امید وارم که خوشتون بیاد .
بازی های منصفانه
بازی ها به دو دسته تقسیم می شوند . بازی های پارتیزانی و بازی های منصفانه . بازی های پارتیزانی مانند شطرنج ، گو ، چکرز یا تخته نرد هستند . اما بازی منصفانه به بازی ای می گوییم که مستقل از اینکه نوبت با چه کسی است ، امکانات یکسانی برای دو طرف وجود داشته باشد . اساس بازی های منصفانه بازی نیم است که این بازی به وسیله ی چند کپه ی لوبیا بازی می شود و هر کس در نوبت خود تعدادی (هر تعداد دلخواه) لوبیا از یکی از کپه ها بر می دارد(حد اقل یک لوبیا) . در این بازی و همه ی بازی های منصفانه بازنده کسی است که نتواند حرکتی را انجام دهد . اگر شما بلد باشید که در این بازی چگونه بازی کنید تا نبازید می توانید در همه ی بازی های منصفانه برنده شوید و فقط کافی است که رابطه ای بین بازی ها و کپه های لوبیا بیابید .
حال به چگونه بازی کردن در بازی نیم می پردازیم .
فرض کنید تنها یک کپه وجود داشته باشد . شما کل کپه را بر می دارید و برنده می شوید . پس یک کپه لوبیا وضعیت برد است . حال فرض کنید دو کپه لوبیا در اختیار داشته باشید . اگر تعداد لوبیا در کوپه ها برابر نباشد ، شما کوپه ی بیشتر را هم اندازه با کوپه ی کوچکتر می کنید و از آن پس از اصل مشابه سازی استفاده می کنید . یعنی مثلا اگر حریف شما n لوبیا از یک کپه بردارد شما همان تعداد لوبیا را از کوپه ی دیگر بر می دارید . به سادگی می شود دید که شما برنده می شوید . پس دو کپه لوبیای نامساوی وضعیت برد و دو کپه ی مساوی وضعیت باخت را دارند .
حال اگر تعداد کپه ها بیشتر از دو بود باید چه کار کنیم . عددی که بیانگر تعداد لوبیاهای هر کپه است را در مبنای دو می نویسیم . فرض کنید که ما 4 کپه با اندازه های 27 ، 23 ، 22 و 15 داریم . بست آنها در مبنای 2 به ترتیب برابر 11011 ، 10111 ، 10110 ، 01111 می شود . ( رقم اول از سمت راست برابر یک ، رقم دوم برابر 2 ، رقم سوم 4 و ...) این اعداد را زیر هم می نویسیم . مشاهده می شود که سه تا رقم 1 در جایگاه 16 ، 2 تا در 8 ، 3 تا در 4 ، 4 تا در 2 و 3 تا در 1 وجود دارند . کاری که می باید بکنیم این است که تعداد همه ی این دسته ها را زوج کنیم (حرکت مطلوب می تواند برداشتن 21 لوبیا از کپه ی 23 تایی باشد). حریف ما مجبور است زوجیت را از بین ببرد و ما دوباره کپه ها را اصلاح می کنیم و اگر این کار را ادامه دهیم می بینیم که به برد ما می انجامد . پس وضعیت های برد در نیم دقیقا آنهایی هستند که هر توانی از 2 ، به تعداد دفعات زوج ظاهر می شود .
جمع نیم : جمع در مبنای دو بدون رقم انتقالی ( این یک عمل منطقی است که اغلب در کامپوتر های خیلی کوچک و ماشین حساب ها وجود دارد و آن را احتمالا XOR یعنی یای انحصاری می نامند .)
طبق تعریف داریم : وضعیت های باخت در نیم دقیقا آنهایی هستند که کپه های نیم در آنها اندازه هایی دارند که مجموع نیمشان صفر می شود .
می بینیم که می توان با روش بالا در بازی نیم برنده شد . برای برنده شدن در سایر بازی های منصفانه باید در هر وضعیت کپه نیم معادل را بدست آورید .
اگر مایل بودید که بازی های بیشتری رو یاد گرفته و طریقه ی بدست آوردن مقدار نیم در آن ها را بدانید می توانید اعلام کنید تا بیشتر به مبحث نظریه ی بازی ها بپردازم .

m_honarmand_j
29-01-2007, 00:09
سلام
دوستان اگر نظری دارید ، اگر پیشنهاد یا انتقادی دارید ، اگر مایلید در موضوع معیینی سوال مطرح بشه ، اگر ... لطفا بهم اطلاع بدید . ممنون می شم .

m_honarmand_j
29-01-2007, 19:36
سلام
K رنگ در اختیار داریم . با چند روش می توان ضلع های یک n ضلعی منتظم را رنگ کرد به نحوی که ضلع های مجاور رنگ های مختلف داشته باشند ؟ (چند ضلعی را نمی توان چرخاند)

سلام
مسئله را بر اساس زوج یا فرد بودن K حل کنید . اولین ضلع کی می خواهیم رنگ کنیم به K روش رنگ می شود . ضلع مجاور آن با K-1 رنگ و ضلع مجاور ضلع دوم نیز با K-1 رنگ و ...:sad:

m_honarmand_j
29-01-2007, 19:38
سلام
در مربع 5*5 ، 16 خانه را رنگ کرده ایم . ثابت کنید ، می توان در آن ، یک مربع 2*2 پیدا کرد که دست کم سه خانه ی آن ، رنگ شده باشد .

سلام
فرض می کنیم این طور نباشد . ستون سمت راست و سطر پایین را در نظر می گیریم . بدیهی است که باید دست کم 8 خانه ی آن ها رنگ شده باشد ( و البته نه هر 9 خانه) و ، در نتیجه ، باید خانه هایی از جدول که پس از حذف 9 خانه ی گفته شده در ستون سمت راست و سطر پایین قرار می گیرند ، بدون رنگ باشند . در نتیجه در مربع 3*3 گوشه ی چپ و بالای جدول ، باید 8 خانه ی رنگی داشته باشیم ، و این ، به معنای آن است که در آن جا مربع 2*2 با سه خانه یا چهار خانه ی رنگی وجود دارد.

m_honarmand_j
30-01-2007, 01:54
سلام
دوستان ، به علت نزدیک شدن امتحان المپیادم تا مدتی نمی تونم به طور منظم سوال بزارم ولی هر وقت تونستم این کار و می کنم . شما به بزرگواری خودتون ببخشید .

m_honarmand_j
30-01-2007, 18:12
سلام
صفحه کاغذ شطرنجی بی پایانی را با 9 رنگ مختلف رنگ کرده ایم . هر خانه یک رنگ دارد و در ضمن از همه ی رنگ ها استفاده شده است . دو رنگ را مجاور می نامیم هرگاه دوخانه که با یک ضلع مشترک پیدا شود که با این دو رنگ ، رنگ شده باشند . حداقل تعداد زوج رنگ های مجاور چند تاست؟

پاسخ 8 می شود . سعی کنید در حالت کلی ثابت کنید اگر صفحه شطرنجی را با K رنگ رنگ کنیم حداقل K-1 زوج رنگ (یکسان)مجاور هم می شوند .

m_honarmand_j
30-01-2007, 20:53
سلام
یال های یک گراف کامل را که 2*n+1 راس دارد ، با سه رنگ مختلف رنگ کرده ایم. ثابت کنید ، می توان یکی از رنگ ها و n+1 راس را طوری انتخاب کرد که ، از هر یک از این راس ها به هر راس دیگر از آن ها ، بتوان ار طریق یال هایی حرکت کرد که دارای رنگ انتخابی ما باشند .

سلام
استقرای روی n . مجموعه ی X شامل n راس را انتخاب می کنیم که با یال های (مثلا با رنگ اول) به هم مربوط اند . اکنون یکی از آن ها را کنار می گذاریم و از بقیه راس ها ، باز هم n راس را انتخاب می کنیم که با یک رنگ به هم مربوط باشند . ( مجموعه ی Y) .
حالت اول . این رنگ ، همان رنگ اول است . در این صورت ، دو انتخاب از n راس ، نباید متقاطع باشند و راس باقی مانده ی A ، به این 2n راس تنها با یال های به رنگ دوم و سوم ، به هم مربوط اند . ولی در این صورت تعداد یال های یکی از این دو رنگ ، دست کم برابر با n ؛ و n راس متناظر ، همراه با A ، n+1 راس مورد نظر را تشکیل می دهند .
حالت دوم . رنگ اخیر ، رنگ دوم ( یا سوم ) است . دراین حالت ، همه ی 2n+1 راس را به دو دسته تقسیم می کنیم : دسته ی اول شامل سه راس که در یکی از دو مجموعه ی X یا Y باشند ؛ دسته ی دوم ، شامل بقیه ی راس ها . به سادگی دیده می شود ، به جز حالت بی معنی که ممکن است پیش بیاید ، هر یک از این دسته ها به دو بخش چنان تقسیم می شود که ، راس های بخش های مختلف دسته ، تنها با یال های به رنگ سوم (و یا دوم) می توانند به هم مربوط شده باشند ؛ در غیر این صورت ، مجموعه ی شامل n+1 راس ( که مورد نظر ماست) ، بلا فاصله ظاهر می شود . در یکی از این دسته ها ، دست کم n+1 راس وجود دارد که تنها با یال های به رنگ سوم به هم مربوط اند : همان چیزی که لازم داریم .

m_honarmand_j
30-01-2007, 20:57
سلام
راس های یک گراف محدود با دورنگ مختلف رنگ شده اند . در هر ثانیه ، هر نقطه تغییر رنگ می دهد و به رنگی در می آید که در همسایگی آن بیشتر است . ثابت کنید ، برای هر نقطه لحظه ای فرا می رسد که ، بعد از آن ، یا تغییر رنگ نمی دهد و یا در هر ثانیه ، تغییر رنگ می دهد .

سلام
این مسئله یکی از مسائل واقعا جالب در مبحث گراف بود که دیدم . من که از حل این مسئله لذت بردم .امید وارم شماهم خوشتون بیاد .
فرض کنید که این طور نباشد . در نتیجه راسی وجود دارد که گاهی متناوبا رنگش عوض می شود و گاهی رنگش ثابت می ماند . فرض کنید که این راس زوج همسایه دارد که متناوبا رنگشان عوض می شود . در این صورت یا از هر رنگ در همسایه های آن به تعداد مساوی قرار دارد و یا تعداد یک رنگ از دیگری بیشتر است . در حالت اول همیشه باید رنگ راس مفروض ثابت بماند و در حالت دوم همیشه باید رنگ راس مفروض عوض شود و این خلاف فرض ماست . به سادگی دیده می شود اگر تعداد همسایه های راس ما فرد با شد و همگی متناوبا عوض شوند باز هم خواسته ی ما بر آورده نمی شود . همین طور حالتی که همسایه از داشته باشد که رنگش ثابت باشد نیز رد می شود (یکم پیچیده تر از حالت های قبل) در نتیجه باید راسی در همسایگی راس اول v1 مانند v2 وجود داشته باشد که آن هم مانند v1 تغییر رنگ دهد یعنی گاهی اوقات ثابت و گاهی متناوب . اگر v2 همسایه ی دیگری از این گونه نداشته باشد پس تغییر رنگ آن وابسته به راس v1 می شود و باز خواسته برآورده نمی شود . در نتیجه این راس نیز باید همسایه ای مانند v3 داشته باشد که تغییر رنگ آن نیز همانند v2 گاهی متغییر و گاهی ثابت باشد . و آن راس نیز باید همسایه ی دیگری داشته باشد ... . اگر ما در مرحله ای به راسی تکراری برسیم (یک دور تشکیل شود) می بینیم که تغییر رنگ همه ی راس ها وابسته به هم در نتیجه خواسته ی ما بر آورده نمی شود . چون گراف ما متناهی است پس نمی تواند این رویه تا بینهایت ادامه داشته باشد و چون تعداد دور ها در گراف متناهی است سر انجام ما باید در یکی از این دور ها بی افتیم و یا به یک برگ برسیم که در هر دو صورت نمی توان خواسته ی خود را بر آورده کنیم در نتیجه فرض ما اشتباه بوده و چنین راسی نداریم . :blink:

m_honarmand_j
06-02-2007, 16:27
سلام
فرض کنید یک میلیارد ( کمتر هم می شه ) ستاره در آسمان وجود دارد . ثابت کنید در بین آن ها حداقل 79 فاصله ی متفاوت وجود دارد .

m_honarmand_j
06-02-2007, 16:28
سلام
یک دنباله به طول 1000 داریم که در آن هزار عدد آمده اند .(لزوما متفاوت نیستند) در هر مرحله زیر هر عدد تعداد باری که آن عدد در دنباله ظاهر شده است را می نویسیم . و دوباره روی دنباله ی به دست آمده عمل بالارا انجام می دهیم . ثابت کنید به مر حله ای می رسیم که بعد از آن دنباله تکرار می شود .

m_honarmand_j
06-02-2007, 16:35
سلام
یک اژدها 100 سر دارد و یک شوالیه می تواند با هر ضربه ی شمشیر خود 15 یا 17 یا 21 یا 5 سر اژ دها را قطع کند و برای این حالت به ترتیب 24 و 2 و 15 و 17 سر جدید از بدن اژدها رشد می کند . اگر تمام سرهای اژدها قطع شود اژدها خواهد مرد . آیا ممکن است اژدها بمیرد؟

m_honarmand_j
06-02-2007, 17:25
سلام
خوب اینجوری بهتره ولی جواب سوال هنوز غلطه!

سلام
من قبلا با دو رنگ مسئله رو حل کردم ولی هرچی فکر کردم یادم نیومد چه جوری ولی با سه رنگ می شه . با سه رنگ 1و2و3 جدول و رنگ می کنیم . سطر پایین از سمت چپ شروع می کنیم و به ترتیب به سمت راست میرویم و به ترتیب رنگ های 1و2و3 و در خانه ها قرار می دهیم . در سطر بالا همین کار را از رنگ 2 شروع می کنیم و در سطر بالای آن از رنگ 3 شروع می کنیم . مشاهره می شود که در هر گام از خانه ی 1 به خانه ای بارنگ 2 و از رنگ 2 به خانه ای به رنگ3 و از 3 به 1 می رویم . چون 22 خانه به رنگ 2 و 21 خانه از 1 و21خانه از داریم 3 پس یک خانه به رنگ 2 باقی می ماند .

m_honarmand_j
11-02-2007, 01:32
سلام
یک اژدها 100 سر دارد و یک شوالیه می تواند با هر ضربه ی شمشیر خود 15 یا 17 یا 21 یا 5 سر اژ دها را قطع کند و برای این حالت به ترتیب 24 و 2 و 15 و 17 سر جدید از بدن اژدها رشد می کند . اگر تمام سرهای اژدها قطع شود اژدها خواهد مرد . آیا ممکن است اژدها بمیرد؟


سلام
خیر .مشاهده می شود که با اعمال گفته شده باقی مانده تعداد سر های اژدها به 3 همیشه ثابت می ماند . در اول این باقی مانده برابر 1 است پس هیچ گاه اژدها نمی میرد . :blink:

m_honarmand_j
11-02-2007, 01:34
سلام
یک دنباله به طول 1000 داریم که در آن هزار عدد آمده اند .(لزوما متفاوت نیستند) در هر مرحله زیر هر عدد تعداد باری که آن عدد در دنباله ظاهر شده است را می نویسیم . و دوباره روی دنباله ی به دست آمده عمل بالارا انجام می دهیم . ثابت کنید به مر حله ای می رسیم که بعد از آن دنباله تکرار می شود .

سلام
از اکسترمال استفاده کنید و ثابت کنید هر ستون از مرحله ی دوم به بعد حداقل دوبرابر می شود . و یک مرز بالایی برای اعداد در نظر بگیرید . ثابت می شود همه ی دنباله ها بعد از حداکثر 11 مرحله متناوب می شوند. :happy:

m_honarmand_j
11-02-2007, 01:37
سلام
فرض کنید یک میلیارد ( کمتر هم می شه ) ستاره در آسمان وجود دارد . ثابت کنید در بین آن ها حداقل 79 فاصله ی متفاوت وجود دارد .

سلام
اول بیاد مسوله رو در حالتی حل کنید که مثلا 500 نقطه در یک صفحه باشند و دو نقطه که بشترین فاصله رو دارند در نظر بگیرید و به مرکز یکی از آنها و به شعاع فاصله ی آنها دایره ای بزنید و ...
اگر توانیتسد به نتیجه ای برسید بیاید همین کار را در فضا انجام بدهید و چیزی را که در اول بدست می آورید را به فضای سه بعدی تامیم دهید .

m_honarmand_j
11-02-2007, 01:51
سلام
یک مربع n*n داریم . در هر مرحله می توانیم یک مربع m*m انتخاب کنیم و تمام اضلاع محیط آن را رنگ کنیم . حد اقل چند مرحله نیاز است تا کل جدول را رنگ کنیم ؟

m_honarmand_j
11-02-2007, 01:57
سلام
n اتومبیل در یک مسیر دایره ای وجود دارند . تمام آنها با هم مجموعا به اندازه ای سوخت دارند که یک اتومبیل با آن بتواند یک مسیر را به طور کامل طی کند . نشان دهید اتومبیلی وجود دارد که می تواند تمام مسیر را یک بار طی کند در صورتی که از اتومبیل های دیگر سوخت بگیرد .

m_honarmand_j
15-02-2007, 02:51
سلام
یک مربع n*n داریم . در هر مرحله می توانیم یک مربع m*m انتخاب کنیم و تمام اضلاع محیط آن را رنگ کنیم . حد اقل چند مرحله نیاز است تا کل جدول را رنگ کنیم ؟

سلام
حد اقل 2n-2 مرحله لازم است . (فقط برای مربع 2 در 2 باید حداقل سه مرحله انجام داد .) می توان تعدادی پاره خط خاص را در نظر گرفت که در هر مرحله حداکثر می توان دو تا از آنها را در هر مرحله انتخاب کرد . (سعی کنید این پاره خط ها را بیابید) می توان روشی نیز ارائه داد که با تعداد مراحل گفته شده بتوان کل جدول را رنگ کرد . از گوشه های سمت چپ بالا و سمت راست پایین مربع هایی به صورت( n-(2k-1 انتخاب کرد و از گوشه های مخالف مربع هایی به صورت n-2k انتخاب کرد که در آن ها k بین 1 و n است . :cool:

m_honarmand_j
15-02-2007, 03:04
سلام
n اتومبیل در یک مسیر دایره ای وجود دارند . تمام آنها با هم مجموعا به اندازه ای سوخت دارند که یک اتومبیل با آن بتواند یک مسیر را به طور کامل طی کند . نشان دهید اتومبیلی وجود دارد که می تواند تمام مسیر را یک بار طی کند در صورتی که از اتومبیل های دیگر سوخت بگیرد .

سلام
بر روی تعداد اتومبیل ها استقرا می زنیم . به ازای n مساوی 1 شرط برقرار است . یعنی این یک ماشین به اندازه ای سوخت دارد که بتواند مسیر را بپیماید . فرض می کنیم به ازای n-1 ماشین شرط برقرار باشد . حال n ماشین را فرض می کنیم . باید ماشینی وجود داشته باشد تا بتواند خورش را به ماشین بعدی در مسیر برساند . ( اگر چنین ماشینی وجود نداشته باشد در نتیجه مجموع سوخت آنها به اندازه ای نیست که یک ماشین با آن بتواند کل مسیر را طی کند) این ماشین خود را به ماشین بعدی می رساند و سوخت ماشین بعدی را می گیرد . حال طبق فرض استقرا این n-1 ماشین باقی مانده می توانند خواسته ی مسئله را برآورده کنند .

m_honarmand_j
15-02-2007, 03:13
سلام
دو نفر روی یک جدول 10*10 این چنین بازی می کنند . مهره ای را روی گره مرکز صفه گذاشته اند و هر کس در نوبت خود مهره را به گره های مجاور حرکت می دهد به شرطی که طول حرکت او نسبت به حرکت بازیکن قبلی بیشتر باشد. کسی می بازد که نتواند حرکتی انجام دهد . اگر هر دوبازیکن بهترین حرکت را انجام دهند چه کسی می تواند طوری بازی کند که ببرد؟

m_honarmand_j
18-02-2007, 02:09
سلام
دو نفر روی یک جدول 10*10 این چنین بازی می کنند . مهره ای را روی گره مرکز صفه گذاشته اند و هر کس در نوبت خود مهره را به گره های مجاور حرکت می دهد به شرطی که طول حرکت او نسبت به حرکت بازیکن قبلی بیشتر باشد. کسی می بازد که نتواند حرکتی انجام دهد . اگر هر دوبازیکن بهترین حرکت را انجام دهند چه کسی می تواند طوری بازی کند که ببرد؟

سلام
نفر دوم استراتژی برد دارد. هر حرکتی که نفر اول انجام دهد نفر دوم قرینه ی آن حرکت را نسبت به مرکز انجام می دهد .

m_honarmand_j
18-02-2007, 15:38
سلام
سه حشره رو یک خط راست اند . آن ها مرتب از روی یکدیگر می پرند( هر حشره از روی یک حشره نه دو حشره). آیا ممکن است ، بعد از 1985 پرش در جای اول خود باشند؟

m_honarmand_j
18-02-2007, 15:42
سلام
روی دو سکو n صندوق با شماره های 1 تا n به طور نامنظم و دلخواه قرار دارند با جرثقیل می توان هر بار تعدادی صندوق را از یک سکو به سکوی دیگر منتقل کرد . ثابت کنید با انجام 2n-1 بار عمل جرثقیل می توان همه ی صندوق ها را در یک سکو به ردیف شماره های آن ها گذاشت.

m_honarmand_j
19-02-2007, 01:00
سلام
وجوه یک مکعب را می خواهیم با دو رنگ رنگ کنیم به طوری که از هر رنگ حداقل یک بار استفاده شده باشد . به چند طریق می توان این کار را انجام داد ؟ ( حالت هایی که با گردش نسبت به هم بدست می آیند را یک بار در نظر بگیرید)

m_honarmand_j
19-02-2007, 01:09
سلام
در مسابقات کشتی پهلوانی 9 نفر دو به دو با هم مسابقه داده اند . حداکثر چند نفر بیش از 4 مسابقه را برده اند؟
(المپیاد ریاضی مرحله ی اول سال 85)

m_honarmand_j
20-02-2007, 03:51
سلام
ثابت کنید می توان ناحیه های ایجاد شده به وسیله ی n خط که هیچ دوتایی موازی و هیچ سه تایی هم راس نیستند
را با دو رنگ چنان رنگ کرد که هیچ دو ناحیه ی مجاور همرنگ نباشند .

m_honarmand_j
20-02-2007, 03:56
سلام
ثابت کنید می توان ناحیه های ایجاد شده توسط n دایره ی غیر تودرتو( یعنی هر دو دایره همدیگر را در دو نطقه قطع می کنند) را با دورنگ چنان رنگ کرد که هیچ دو ناحیه مجاوری رنگ یکسان نداشته باشند.

m_honarmand_j
20-02-2007, 03:59
سلام
ثابت کنید که هر n خط در صفحه که هیچ دو خطی موازی نیستند و هیچ سه تایی از یک نقطه نمی گذرند چند ناحیه ایجاد می کنند .

m_honarmand_j
20-02-2007, 04:02
سلام
ثابت کنید که هر n خط در صفحه که هیچ دوتایی موازی و هیچ سه تایی از یک نقطه نمی گذرند حداقل n-2 تا مثلث غیر متداخل (در همدیگر قرار ندارند) ایجاد می کنند .

m_honarmand_j
20-02-2007, 04:05
سلام
ثابت کنید مجموع اعداد 1 تا n برابر n*(n+1)/2 است .

attractive_girl
21-02-2007, 20:33
سلام .تو روخدا ایندفعه دستم خیلی گیرتونه خواهشا سوالم رو جواب بدین .اگه جواب بدین ثواب میکنین .
اثبات حالت دوم تشابه ها یعنی حالت متناسب بودن 3ضلع در مثلث رو میخوام.(در حد دوم دبیرستان)

ironroad
22-02-2007, 08:19
با استفاده از اصل لانه کبوتری اثبات کنید

به ازای هر عدد طبیعی حداقل یک مضرب وجود دارد که فقط از اعداد 0 و 1 تشکیل شده مثلا

عدد 4 مضرب 100 دارد یا عدد 2 مضرب های 10 و 100 و 110 و 1010 و ... دارد

m_honarmand_j
23-02-2007, 16:47
سلام
امروز امتحان المپیاد کامپیوتر مرحله ی اول و دادم . هفته ی قبل هم امتحان ریاضی بود . به احتمال زیاد تو هردوتاشون در بیام . (شما ها هم دعام کنید)
از این به بعد هم سعی می کنم سوال های جالبتری و براتون بزارم .
امروز که دیدم دونفر از دوستان در بحث شرکت کردند خیلی خوشحال شدم . امیدوارم که این شرکتشون ادامه پیدا کنه .

Moh3en_DDD
23-02-2007, 17:29
سلام
ثابت کنید مجموع اعداد 1 تا n برابر n*(n+1)/2 است .

s = 1+2+3+...+n
s = n+(n-1)+(n-2)+(n-3)+...+1
+__________________________
2s = (n+1)+(n+1)+(n+1)+...+(n+1)

==> 2s = n*(n+1) ==> s = n*(n+1) /2

m_honarmand_j
23-02-2007, 17:32
سلام .تو روخدا ایندفعه دستم خیلی گیرتونه خواهشا سوالم رو جواب بدین .اگه جواب بدین ثواب میکنین .
اثبات حالت دوم تشابه ها یعنی حالت متناسب بودن 3ضلع در مثلث رو میخوام.(در حد دوم دبیرستان)

[B]سلام
اثبات تشابه دو مثلث هنگامی که سه ضلع مثلثی با سه ضلع مثلث دیگر متناسب باشد:
دو مثلث ABC و 'A'B'C را در نظر بگیرید ( فرض کنید ABC مثلث بزگتر است) . روی اضلاع AB و AC در مثلث ABC هم اندازه ی 'A'B و 'A'B جدا کرده و نقاط بدست آمده را "B و "C می نامیم و آنها را به هم وصل می کنیم .
بنا بر فرض داریم : A'B'/AB = A'C'/AC نتیجه : AB"/AB = AC"/AC نتیجه (عکس قضیه تالس) B"C" || BC
از بالا نتیجه می گیریم که زاویه ی C"=C و B"=B .
همچنین بنا بر قضیه ی تالس نتیجه می گیریم که : AB"/AB = AC"/AC = B"C"/BC
از طرفی : A'C'/AC = B'C'/BC
از دو خط بالا می توان نتیجه گرفت :
B"C"/BC = B'C'/BC then B"C" = B'C' then ABC~AB"C" =>A^=A'^ , B"^=B'^ , C"^= C'^ and
و بنا به تساوی ای که در بالا داشتیم (تساوی زاویه ها) زاویه های : ^A'^=A^ , B'^=B^ , C'^=C باهم مساوی اند .
اثبات به پایان رسید . [/B
اگه در جایی بد توضیح دادم بگید تا دوباره توضیح بدم .(ببخشید منظورتون از ایندفعه دستم خیلی گیرتونه یعنی چی ؟ مگه قبلا هم دستتون گیر من بوده؟)

m_honarmand_j
23-02-2007, 17:36
با استفاده از اصل لانه کبوتری اثبات کنید

به ازای هر عدد طبیعی حداقل یک مضرب وجود دارد که فقط از اعداد 0 و 1 تشکیل شده مثلا

عدد 4 مضرب 100 دارد یا عدد 2 مضرب های 10 و 100 و 110 و 1010 و ... دارد

سلام
الان جوابش یادم نیست . باید حلش کنم . هر وقت حلش کردم حتما جوابشو می زارم . (خیلی زود)

m_honarmand_j
23-02-2007, 17:40
s = 1+2+3+...+n
s = n+(n-1)+(n-2)+(n-3)+...+1
+__________________________
2s = (n+1)+(n+1)+(n+1)+...+(n+1)

==> 2s = n*(n+1) ==> s = n*(n+1) /2

سلام
این هم یک راه حل این مسئله است . ببینید می تونید با استقرا حلش کنید .

Moh3en_DDD
23-02-2007, 17:43
1377 بردار غیر صفر در صفحه داریم که همه آنها موازی نیستند . این بردار ها چنان اند که هریک از آنها را می توان به صورت مضربی از حاصل بردار های دیگر نوشت . ] مثلا برای بردار V1 عددی مانند K1 هست که V1 = K1 (V1+V2+V3+...+Vn) [

تحت این شرایط ثابت کنید حاصل جمع تمام این بردار ها صفر است .

Moh3en_DDD
23-02-2007, 17:55
A و B و C هر کدام تعدادی کارت پیش خود دارند . اول A از مهره های خود آن قدر به B و C می دهد تا تعداد کارت های هرکدام 2 برابر شود . بعد B هم همین کار را می کند و بعد هم C .

در پایان کارت هر کدام 24 کارت دارند . در ابتدای کار هرکدام چند مهره داشته اند ؟

m_honarmand_j
24-02-2007, 13:07
سلام
ثابت کنید می توان ناحیه های ایجاد شده به وسیله ی n خط که هیچ دوتایی موازی و هیچ سه تایی هم راس نیستند
را با دو رنگ چنان رنگ کرد که هیچ دو ناحیه ی مجاور همرنگ نباشند .

سلام
روی n استقرا می زنیم . برای n=1 که واضح است ( یک ناحیطه را با سیاه و طرف دیگر را با سفید رنگ می کنیم) .
فرض می کنیم برای m<n-1 می توان این کار را کرد . برای n ثابت می کنیم که می توان این کار را کرد .
خط n ام را در نظر نمی گیریم و ناحیه ها را طبق فرض استقرا با دو رنگ رنگ می کنیم . حال خط n ام را اضافه می کنیم . تمام ناحیه هایی که در یک طرف این خط می افتند را رنگشان را عوض می کنیم . می توان ملاحظه کرد که همچنان شرط مسئله برقرار است . (چرا؟) :tongue:

m_honarmand_j
24-02-2007, 13:10
سلام
ثابت کنید می توان ناحیه های ایجاد شده توسط n دایره ی غیر تودرتو( یعنی هر دو دایره همدیگر را در دو نطقه قطع می کنند) را با دورنگ چنان رنگ کرد که هیچ دو ناحیه مجاوری رنگ یکسان نداشته باشند.


سلام
روی n استقرا می زنیم . (پایه ی استقرا را خودتان بررسی کنید) فرض کنید برای کمتر از n شرط مسئله برقرار باشد . حال n دایره را در نظر می گیریم . دایره ی n ام را در نظر نمی گیریم و صفحه را رنگ می کنیم . حال دایره ی n ام را اضافه می کنیم . تمام ناحیه هایی که در داخل این دایره قرار می گیرند را رنگشان را عوض می کنیم . ملاحظه می شود که همچنان شرط مسئله برقرار است (چرا؟) .

m_honarmand_j
24-02-2007, 13:17
سلام
ثابت کنید که هر n خط در صفحه که هیچ دو خطی موازی نیستند و هیچ سه تایی از یک نقطه نمی گذرند چند ناحیه ایجاد می کنند .

سلام
روی n استقرا بزنید و ثابت کنید که هر اگر خط m ام را رسم کنیم ، m ناحیه جدید به ناحیه های قبلی اضافه می کند .
در نتیجه مجموع ناحیه ها می شود : n*(n+1)/2 +1

attractive_girl
24-02-2007, 13:59
[B]سلام
اثبات تشابه دو مثلث هنگامی که سه ضلع مثلثی با سه ضلع مثلث دیگر متناسب باشد:
دو مثلث ABC و 'A'B'C را در نظر بگیرید ( فرض کنید ABC مثلث بزگتر است) . روی اضلاع AB و AC در مثلث ABC هم اندازه ی 'A'B و 'A'B جدا کرده و نقاط بدست آمده را "B و "C می نامیم و آنها را به هم وصل می کنیم .
بنا بر فرض داریم : A'B'/AB = A'C'/AC نتیجه : AB"/AB = AC"/AC نتیجه (عکس قضیه تالس) B"C" || BC
از بالا نتیجه می گیریم که زاویه ی C"=C و B"=B .
همچنین بنا بر قضیه ی تالس نتیجه می گیریم که : AB"/AB = AC"/AC = B"C"/BC
از طرفی : A'C'/AC = B'C'/BC
از دو خط بالا می توان نتیجه گرفت :
B"C"/BC = B'C'/BC then B"C" = B'C' then ABC~AB"C" =>A^=A'^ , B"^=B'^ , C"^= C'^ and
و بنا به تساوی ای که در بالا داشتیم (تساوی زاویه ها) زاویه های : ^A'^=A^ , B'^=B^ , C'^=C باهم مساوی اند .
اثبات به پایان رسید . [/B
اگه در جایی بد توضیح دادم بگید تا دوباره توضیح بدم .(ببخشید منظورتون از ایندفعه دستم خیلی گیرتونه یعنی چی ؟ مگه قبلا هم دستتون گیر من بوده؟)
سلام واقعا ممنون منظورمخیلی کمکم کردین. از اینکه گفتم دستم گیرتونه با 1شخص نبودم آخه دفعه ی قبل کلی سوال نوشتم ولی هیشکی جواب نداد به همین دلیل ناامید شدم ولی حالا کلی شارژ شدم .ممنون تو تایپک فیزیک هک اگه میشه بیا 1سوال اونجا هم دارم اگه بتونی جواب بدی ممنون میشم.در ضمن من هم المپیاد کامپیوتر شرکت کردم ولی با سوالاش آشنایی نداشتم جوابشم خدا داند...

m_honarmand_j
24-02-2007, 14:22
سلام
دوستان همان طور که قبلا گفتم من برای المپیاد تلاش می کنم و مرحله ی اول المپیاد و خیلی خوب دادم :rolleye: . در نتیجه از این به بعد باید به طور فشرده ای خودم و برای مرحله ی دوم آماده کنم . به همین دلیل ممکن است کمتر بتونم برای اتاق ترکیبیات وقت بزارم . ولی سعی می کنم که هر هفته یک بار به اتاق ترکیبیات سر بزنم . اگر دیدید که سوالی پرسیدید و جوابی به شما ندادم از الان از شماها می خواهم که از من دلگیر نشوید . چون واقعا برنامم فشرده است . ممکن است یک یا دو هفته سوالی نذارم و یک باره 10 تا سوال تو اتاق قرار بدم . ممکن هم است که هر روز یک سوال بذارم و کارم اصلا معلوم نیست .
امیدوارم که شماها هم همیشه موفق باشد . اگر مشکلی داشتید بپرسید و من هم سعی می کنم تا حد ممکن جواب شما رو بدم .

m_honarmand_j
24-02-2007, 14:39
سلام واقعا ممنون منظورمخیلی کمکم کردین. از اینکه گفتم دستم گیرتونه با 1شخص نبودم آخه دفعه ی قبل کلی سوال نوشتم ولی هیشکی جواب نداد به همین دلیل ناامید شدم ولی حالا کلی شارژ شدم .ممنون تو تایپک فیزیک هک اگه میشه بیا 1سوال اونجا هم دارم اگه بتونی جواب بدی ممنون میشم.در ضمن من هم المپیاد کامپیوتر شرکت کردم ولی با سوالاش آشنایی نداشتم جوابشم خدا داند...

سلام
خواهش می کنم . قابلی نداشت . من که کاری نکردم .
من نتونستم پیدا کنم که شما تو کدوم قسمت فیزیک سوال پرسیدید . اگر تونستید آدرس دقیق تری بدید حتما سعی می کنم کمکتون بکنم .
امیدوارم که در المپیاد کامپیوتر هم قبول بشید .

m_honarmand_j
24-02-2007, 15:02
سلام
خط شکسته ی بسته ای که پنج ضلع دارد یک پنج ضلعی ستاره ای با زاویه های برابر ساخته است . اگر طول خط شکسته برابر واحد باشد ، محیط پنج ضلعی درونی چیست؟

m_honarmand_j
24-02-2007, 23:09
سلام
یک استاد معبد شائولین پنج نفر از بهترین شاگردانش را به معبد می فرستد تا باهم مبارزه کنند و رتبه بندی شوند . بعد از اینکه مبارزه ی آنها تمام می شود آنها به ترتیب از معبد بیرون می آیند . نفر اول می گوید : من اول شدم . نفر دوم می گوید : من اول نشدم. نفر سوم: من آخر نشدم. نفر چهارم : من نه اول شدم و نه آخر . نفر پنجم : من یا اول شدم یا آخر . استاد پس از خارج شدن آنها می گوید : لی باز هم تو آخر شدی . می دانیم که استاد تمام شاگردانش را می شناسد و آنها دارای نام های مختلف هستند . استاد می داند که چن دروغ گو است و همیشه دروغ می گوید . با این اطلاعات بگویید که لی چندمین نفری بود که از معبد خارج شد .
(مرحله ی اول المپیاد کامپیوتر سال 85)

m_honarmand_j
25-02-2007, 14:02
با استفاده از اصل لانه کبوتری اثبات کنید

به ازای هر عدد طبیعی حداقل یک مضرب وجود دارد که فقط از اعداد 0 و 1 تشکیل شده مثلا

عدد 4 مضرب 100 دارد یا عدد 2 مضرب های 10 و 100 و 110 و 1010 و ... دارد

سلام
یک عدد دلخواه مانند n را در نظر می گیریم . حال اعداد 1و11و111و... را در نظر می گیریم . اگر یکی از آن ها بر n بخشپذیر باشد مسئله حل است . پس فرض می کنیم هیچ کدام بر n بخشپذیر نباشند . بنا به اصل لانه کبوتری حداقل باقی مانده ی دوتا از آن ها بر n مساوی هم است . درنتیجه تفاضل آنها (که تنها از 0 و1 تشکیل شده است) بر n بخشپذیر است . :cool:

m_honarmand_j
25-02-2007, 14:05
سلام
یک مسئله ی جالب:
33 دانش آموز در کلاسی هستند . از هر کس تعداد کسانی در کلاس که اسمشان با اسم او یکی است و همچنین تعداد کسانی را که نام خانوادگی آنها با او یکی است را می پرسیم . در جواب هایی که این دانش آموزان داده اند همه ی اعداد 0 تا 10 آمده است . ثابت کنید در بین این دانش آموزان دونفر وجود دارند که هم نامشان و هم نام خانوادگی شان با هم یکی است .

m_honarmand_j
25-02-2007, 14:30
A و B و C هر کدام تعدادی کارت پیش خود دارند . اول A از مهره های خود آن قدر به B و C می دهد تا تعداد کارت های هرکدام 2 برابر شود . بعد B هم همین کار را می کند و بعد هم C .

در پایان کارت هر کدام 24 کارت دارند . در ابتدای کار هرکدام چند مهره داشته اند ؟

سلام
مراحل کار رو برعکس می کنیم . نفر C را در نظر می گیریم . می خواهیم ببینیم که او چند کارت به A و B داده .
نفر A و B الان 24 تا کارت دارند ، یعنی قبل از این مرحله هر کدام 12 کارت داشته اند . در نتیجه نفر C دارای 48 کارت بوده است . حال نفر B را در نظر می گیریم . می خواهیم ببینیم که او چند کارت به نفر A و C داده است . آنها هر کدام الان به ترتیب دارای 12 و 48 کارت می باشند ، در نتیجه قبل از این مرحله هرکدام به ترتیب دارای 6 و 24 کارت بوده اند و نفر B، دارای 42 کارت بوده است . حال نفر A را در نظر می گیریم . می خواهیم ببینیم که او چند کارت به نفر B و C داده است . آنها هر کدام به ترتیب دارای 42 و 24 کارت هستند . در نتیجه در اول هرکدام به ترتیب دارای 21 و 12 کارت بوده اند و نفر A هم در اول دارای 39 کارت بوده است .
با انجام دادن بازی می بینیم که جواب ها درست هستند . ;)

attractive_girl
25-02-2007, 17:26
سلام
یک استاد معبد شائولین پنج نفر از بهترین شاگردانش را به معبد می فرستد تا باهم مبارزه کنند و رتبه بندی شوند . بعد از اینکه مبارزه ی آنها تمام می شود آنها به ترتیب از معبد بیرون می آیند . نفر اول می گوید : من اول شدم . نفر دوم می گوید : من اول نشدم. نفر سوم: من آخر نشدم. نفر چهارم : من نه اول شدم و نه آخر . نفر پنجم : من یا اول شدم یا آخر . استاد پس از خارج شدن آنها می گوید : لی باز هم تو آخر شدی . می دانیم که استاد تمام شاگردانش را می شناسد و آنها دارای نام های مختلف هستند . استاد می داند که چن دروغ گو است و همیشه دروغ می گوید . با این اطلاعات بگویید که لی چندمین نفری بود که از معبد خارج شد .
(مرحله ی اول المپیاد کامپیوتر سال 85)

من زدم نفر دوم

SRT_71
25-02-2007, 17:57
سلام
یک استاد معبد شائولین پنج نفر از بهترین شاگردانش را به معبد می فرستد تا باهم مبارزه کنند و رتبه بندی شوند . بعد از اینکه مبارزه ی آنها تمام می شود آنها به ترتیب از معبد بیرون می آیند . نفر اول می گوید : من اول شدم . نفر دوم می گوید : من اول نشدم. نفر سوم: من آخر نشدم. نفر چهارم : من نه اول شدم و نه آخر . نفر پنجم : من یا اول شدم یا آخر . استاد پس از خارج شدن آنها می گوید : لی باز هم تو آخر شدی . می دانیم که استاد تمام شاگردانش را می شناسد و آنها دارای نام های مختلف هستند . استاد می داند که چن دروغ گو است و همیشه دروغ می گوید . با این اطلاعات بگویید که لی چندمین نفری بود که از معبد خارج شد .
(مرحله ی اول المپیاد کامپیوتر سال 85)

من فكر مي كنم نفر پنجم باشه
لطفا جواب رو بگيد
ممنون

m_honarmand_j
27-02-2007, 00:57
سلام
سه حشره رو یک خط راست اند . آن ها مرتب از روی یکدیگر می پرند( هر حشره از روی یک حشره نه دو حشره). آیا ممکن است ، بعد از 1985 پرش در جای اول خود باشند؟

سلام
حشره ها را با حروف نشان می دهیم . در تنیجه در اول به صورت A,B,C استاده اند . شش حالت برای استادن آنها وجود دارد(از چپ به راست) :ABC,BCA,CAB,ACB,BAC,CBA .
سه حالت را نوع اول استقرار و سه حالت دوم را نوع دوم استقرار حشره ها می نامیم . به سادگی و با تحقیق معلوم می شود که با هر پرش از نوع اول به نوع دوم و برعکس تبدیل می شود . بنابر این ، بعد از 1985 پرش ، وضع استقرار حشره ها ، با وضع نخستین آنها فرق خواد داشت.

m_honarmand_j
27-02-2007, 01:01
سلام
روی دو سکو n صندوق با شماره های 1 تا n به طور نامنظم و دلخواه قرار دارند با جرثقیل می توان هر بار تعدادی صندوق را از یک سکو به سکوی دیگر منتقل کرد . ثابت کنید با انجام 2n-1 بار عمل جرثقیل می توان همه ی صندوق ها را در یک سکو به ردیف شماره های آن ها گذاشت.

سلام
فرض می کنیم توانسته باشیم در یکی از سکوها k صندوق را به شماره های ردیف (در زیر صندوق شماره 1،روی آن صندوق شماره 2و...وسرانجام در بالا صندوق شماره k) قرار داده باشیم . اکنون ، اگر در روی صندوق شماره k صندوق های دیگری وجود دارد همه ی آنها را به سکوی دیگر منتقل می کنیم و سپس همه صندوق ها را از این سکو (به جز آنها که در زیر صندوق k+1 ام است) به سکوی قبلی و روی صندوق شماره k می بریم . به این ترتیب صندوق با شماره ی (k+1) روی صندوق با شماره ی k قرار می گیرد . روشن است که برای آخرین صندوق یعنی صندوق با شماره ی n بیش از یک جابجایی لازم نیست و بنابر این روی هم با (2n-1) جابجایی به نتیجه ی مطلوب می رسیم.:blush:

m_honarmand_j
27-02-2007, 01:05
سلام
وجوه یک مکعب را می خواهیم با دو رنگ رنگ کنیم به طوری که از هر رنگ حداقل یک بار استفاده شده باشد . به چند طریق می توان این کار را انجام داد ؟ ( حالت هایی که با گردش نسبت به هم بدست می آیند را یک بار در نظر بگیرید)

سلام
فرض کنید که یک وجه با رنگ 1 رنگ شده باشد . می توان مکعب را طوری چرخاند که این وجه در پایین قرار گیرد . پس برای این حالت بک مکعب بیشتر نداریم . حالتی که پنج وجه را رنگ 1 بزنیم انگار یک وجه را با رنگ 2 رنگ کردیم در نتیجه برای این حالت نیز یک مکعب بیشتر نداریم . حال فرض کنید دو وجه را با رنگ 1 رنگ کرده این . می توان مکعب را طوری چرخاند که یک وجه در پایین قرار گیرد . وجه دیگر یا در مقابل این وجه با در یکی از اضلاع با آن مشترک است . در نتیجه برای این حالت دو مکعب داریم . حالت چهار وجه با رنگ 1 و دو وجه با رنگ2 نیز به همین ترتیب است . حال فرض کنید 3 وجه با رنگ 1 و سه وجه با رنگ 2 رنگ شده باشند . می توان مکعب را طوری چرخاند که یک وجه با رنگ 1 در پایین قرار گیرد . برای دو وجه دیگر دو حالت داریم . یا یکی از آنها در روبه روی وجه قبلی و آن یکی با هرکدام از آنها در یک ضلع مشترک باشد یا هر سه وجه در یک گوشه مشترک باشند . در نتیجه کل حالت ها برابر با 8 می شوند .

m_honarmand_j
27-02-2007, 01:06
سلام
در مسابقات کشتی پهلوانی 9 نفر دو به دو با هم مسابقه داده اند . حداکثر چند نفر بیش از 4 مسابقه را برده اند؟
(المپیاد ریاضی مرحله ی اول سال 85)

سلام
در این مسابقات 36 مسابقه انجام شده است (هر کس 8 مسابقه) . اگر 8 نفر 5 مسابقه را برده باشند در نتیجه 8*5=40 مسابقه انجام شده که این خلاف شرایط مسئله است . اگر 7 نفر ایتگونه باشند در نتیجه 7*5=35 مسابقه را انجام داده اند و بقیه مسابقات نیز نتایجی داشته ایند که برای ما مهم نیستند . حال به این می پردازیم که یک چنین حالتی را ارائه دهیم .(هرکس را با کسانی که او از آن ها برده نشان می دهیم)1:2،3،4،6،7 . 2:3،4،5،6،7 . 3:4،5،6،7،8 . 4:5،6،7،8،9 . 5:6،7،8،9،1 . 8:1،2،6،7،9 . 9:1،2،3،6،7 . 7:6 و 6 هیچکس را نبرده است .

m_honarmand_j
27-02-2007, 01:12
سلام
ثابت کنید که هر n خط در صفحه که هیچ دوتایی موازی و هیچ سه تایی از یک نقطه نمی گذرند حداقل n-2 تا مثلث غیر متداخل (در همدیگر قرار ندارند) ایجاد می کنند .

سلام
خط n ام را در نظر بگیرید . هرخط دیگر این خط را در یک نقطه قطع می کند . از یک سمت شروع کنید . دو نقطه ی برخورد اول را در نظر بگیرید و ... :evil: :blink:

m_honarmand_j
27-02-2007, 01:14
سلام
یک استاد معبد شائولین پنج نفر از بهترین شاگردانش را به معبد می فرستد تا باهم مبارزه کنند و رتبه بندی شوند . بعد از اینکه مبارزه ی آنها تمام می شود آنها به ترتیب از معبد بیرون می آیند . نفر اول می گوید : من اول شدم . نفر دوم می گوید : من اول نشدم. نفر سوم: من آخر نشدم. نفر چهارم : من نه اول شدم و نه آخر . نفر پنجم : من یا اول شدم یا آخر . استاد پس از خارج شدن آنها می گوید : لی باز هم تو آخر شدی . می دانیم که استاد تمام شاگردانش را می شناسد و آنها دارای نام های مختلف هستند . استاد می داند که چن دروغ گو است و همیشه دروغ می گوید . با این اطلاعات بگویید که لی چندمین نفری بود که از معبد خارج شد .
(مرحله ی اول المپیاد کامپیوتر سال 85)

سلام
فرض کنید نفر اول و نفر آخر هر دو راستگو باشند . در نتیجه نفر اول در مسابقه اول شده و نفر آخر نیز آخر شده است . حال نفر دوم را در نظر بگیرید . او نمی تواند دروغگو باشد چون در این صورت باید اول شده باشد . در نتیجه او راست گفته است . نفر سوم را در نظر بگیرید . او نیز نمی تواند دروغگو باشد چون در این صورت او آخر می شود . نفر چهارم را در نظر بگیرید . اگر او دروغ گو باشد در نتیجه هم باید اول شده باشد و هم آخر . در نتیجه نفر اول و نفر پنجم نمی توانند هر دو راستگو باشند . حال فرض کنید نفر اول دروغگو باشد . در نتیجه او آخر شده و ما می دانیم که چن دروغ گو است ولی لی آخر شده است در نتیجه نفر اول نیز نمی تواند دروغگو باشد . پس نفر پنجم دروغگو است . درنتیجه نفر اول که خارج شده اول شده است، نفر پنجم نیز نه اول شده و نه آخر . تنها کسی که می تواند آخر شده باشد نفر دوم است زیرا او گفته که اول نشده در نتیجه بنا به گفته ی او و سایرین تنها او می تواند آخر شده باشد .;) :tongue:

m_honarmand_j
27-02-2007, 01:46
سلام
دوستان اگر در مورد سوالات المپیاد کامپیوتر مرحله ی اول امسال مشکلی داشتید می تونید بپرسید . تا حد امکان سعی می کنم جواب و بهتون بدم . چون تقریبا به جز چند تا از سوالات بقیه (بیش از 30 تا) رو حل کردم . (اگر در مورد سوالات ریاضی امسال هم سوال داشتید بپرسید)

ali_hp
28-02-2007, 14:40
حال فرض کنید نفر اول دروغگو باشد . در نتیجه او آخر شده و ما می دانیم که چن دروغ گو است ولی لی آخر شده است در نتیجه نفر اول نیز نمی تواند دروغگو باشد . پس نفر پنجم دروغگو است . درنتیجه نفر اول که خارج شده اول شده است، نفر پنجم نیز نه اول شده و نه آخر . تنها کسی که می تواند آخر شده باشد نفر دوم است زیرا او گفته که اول نشده در نتیجه بنا به گفته ی او و سایرین تنها او می تواند آخر شده باشد .;) :tongue:
سلام
من که نفهمیدم چی شد!چرا اگه نفر اول دروغگو باشه نتیجه میشه اخر شده؟

m_honarmand_j
01-03-2007, 18:26
سلام
من که نفهمیدم چی شد!چرا اگه نفر اول دروغگو باشه نتیجه میشه اخر شده؟



سلام
ببخشید در صورت سوال آمده بود که اطلاعات برای یافتن لی کافی است .(وقتی داشتم سوال و جواب و می گذاشتم صورت سوال پیشم نبود)
اگر نفر اول دروغگو باشد هم نفر پنجم می تواند آخر شده باشد و هم نفر دوم که باتوجه با کافی بودن اطلاعات این حالت کنار گذاشته می شود . اگر دوباره مسئله را حل کنید می بینید که فقط نفر دوم می تواند آخر شده باشد .

m_honarmand_j
01-03-2007, 18:36
سلام
یک مسئله ی جالب :
یک ماتریس n*n داریم که از 0و1و1- تشکیل شده است . در هر سطر و هر ستون یک 1 و یک1- وجود دارد . دو نوع عمل داریم . جابجا کردن دو ستون با هم و یا جابجا کردن دو سطر باهم . ثابت کنید می توان با تعدادی حرکات جای 1 ها را با 1- ها عوض کرد . حداقل تعداد حرکت ها را نیز بیابید .

m_honarmand_j
01-03-2007, 18:52
سلام
ثابت کنید یک مستطیل 5*7 را نمی توان با n لایه (همه جا باید n لایه داشته باشد) از ترومینو ها (یک مربع 2*2 که یک گوشه ی آن حذف شده باشد) پوشاند .

m_honarmand_j
01-03-2007, 19:22
سلام
یک سوال سخت (ولی حل می شه):
بیشترین تعداد زیر دنباله به صورت n , n+1, n+2 از دنباله ای به طول 2001 عدد صحیح چیست ؟ برای مثال دنباله ی 1,2,2,3,3 از 5 عدد دارای 4 زیردنباله از آن نوع است .

m_honarmand_j
01-03-2007, 19:25
سلام
به چند طریق می شود n نفر را دور یک میز چید؟

m_honarmand_j
01-03-2007, 19:26
سلام
به چند طریق می شود n مرد را با زن هایشان دور میز چید که هیچ زن و شوهری کنار هم نباشند ؟

m_honarmand_j
01-03-2007, 19:34
سلام
برای آنکه بتوان راس های گرافی را با دو رنگ طوری رنگ کرد که هیچ دو راس همسایه ای رنگ مشترک نداشته باشند حد اکثر چند یال می توان رسم کرد ؟

m_honarmand_j
01-03-2007, 19:37
سلام
شرط لازم و کافی برای آنکه در یک گراف مسیری وجود داشته باشد که از همه ی یال ها و از هر کدام یک بار بگذرد چیست؟

m_honarmand_j
01-03-2007, 19:39
سلام
در یک گراف باید حداقل چند یال داشته باشیم تا بتوانیم یک مثلث در آن پیدا کنیم؟

ali_hp
01-03-2007, 19:54
سلام
ببخشید در صورت سوال آمده بود که اطلاعات برای یافتن لی کافی است .(وقتی داشتم سوال و جواب و می گذاشتم صورت سوال پیشم نبود)
اگر نفر اول دروغگو باشد هم نفر پنجم می تواند آخر شده باشد و هم نفر دوم که باتوجه با کافی بودن اطلاعات این حالت کنار گذاشته می شود . اگر دوباره مسئله را حل کنید می بینید که فقط نفر دوم می تواند آخر شده باشد .
سلام
یه لطفی کن صوت این مساله رو کامل بزار من هنوز نفهمیدم سوال چیه!

m_honarmand_j
01-03-2007, 20:21
استاد معبد شائولین 5 نفر از شاگردانش را به داخل معبد می فرستد تا با هم بجنگند و رتبه بندی شوند . آنها پس از چند روز مسابقه از معبد خارج می شوند و به ترتیب این جملات را می گویند : نفر اول: من اول شدم . نفر دوم : من اول نشدم . نفر سوم : من آخر نشدم . نفر چهارم : من نه آخر شدم و نه اول . نفر پنجم : من یا اول شدم یا آخر .
پس از آنکه همه از معبد خارج می شوند و جملات را می گویند استاد می گوید : لی تو باز هم آخر شدی . لی چندمین نفری بود که از معبد خارج شد؟
می دانیم که چن دروغ گو است و همیشه دروغ می گوید. استاد همه ی شاگردانش را با نام و چهره می شناسد و اشتباه نمی کند. با اطلاعاتی که از گفته ی شاگردانش دریافت می کنیم حتما می توانیم معلوم کنیم که لی چه کسی بوده است .

m_honarmand_j
01-03-2007, 21:09
سلام
یه لطفی کن صوت این مساله رو کامل بزار من هنوز نفهمیدم سوال چیه!

سلام
صورت سوال و گذاشتم . منظور این سوال اینه که این اطلاعاتی که داریم برای فهمیدن اینکه چه کسی لی هست کافی است و حالتی که نتوان جواب را با اطمینان پیدا کرد را خود سوال رد می کند .

m_honarmand_j
02-03-2007, 00:58
سلام
آیا می توان 250 قطعه ی چوبی 1*1*4 را در یک خانه ی 10*10*10 قرار داد؟

m_honarmand_j
02-03-2007, 01:04
سلام
A و B به ترتیب با دومهره ی شاه سیاه و سفید بازی می کنند . هر کس در نوبت خود شاه خود را به یکی از خانه هایی که قبلا توسط یکی از دو شاه اشغال نشده حرکت می دهد . بازنده کسی است که نتواند حرکتی را انجام دهد . در این بازی چه کسی برنده می شود؟

attractive_girl
02-03-2007, 08:37
سلام
فرض کنید نفر اول و نفر آخر هر دو راستگو باشند . در نتیجه نفر اول در مسابقه اول شده و نفر آخر نیز آخر شده است . حال نفر دوم را در نظر بگیرید . او نمی تواند دروغگو باشد چون در این صورت باید اول شده باشد . در نتیجه او راست گفته است . نفر سوم را در نظر بگیرید . او نیز نمی تواند دروغگو باشد چون در این صورت او آخر می شود . نفر چهارم را در نظر بگیرید . اگر او دروغ گو باشد در نتیجه هم باید اول شده باشد و هم آخر . در نتیجه نفر اول و نفر پنجم نمی توانند هر دو راستگو باشند . حال فرض کنید نفر اول دروغگو باشد . در نتیجه او آخر شده و ما می دانیم که چن دروغ گو است ولی لی آخر شده است در نتیجه نفر اول نیز نمی تواند دروغگو باشد . پس نفر پنجم دروغگو است . درنتیجه نفر اول که خارج شده اول شده است، نفر پنجم نیز نه اول شده و نه آخر . تنها کسی که می تواند آخر شده باشد نفر دوم است زیرا او گفته که اول نشده در نتیجه بنا به گفته ی او و سایرین تنها او می تواند آخر شده باشد .;) :tongue:

آخ جون درست زدم............:laughing: :biggrin:

ironroad
02-03-2007, 10:26
ساندویچ فروشی تبلیغ کرده که بیش از 3000 نوع ساندویچ عرضه می کند
اگر هر ساندویچ را بتوان با 5 نوع گوشت و 2 نوع پنیر و یا بدون آنها و با 2 نوع نان به علاوه 3 نوع مخلفات درست کردآیا تبلیغ این ستندویچ فروشی درست بوده؟ فرض بر این است که هر ساندویچ باید نان و حداقل یک نوع گوشت یا پنیر داشته باشد

m_honarmand_j
02-03-2007, 12:47
ساندویچ فروشی تبلیغ کرده که بیش از 3000 نوع ساندویچ عرضه می کند
اگر هر ساندویچ را بتوان با 5 نوع گوشت و 2 نوع پنیر و یا بدون آنها و با 2 نوع نان به علاوه 3 نوع مخلفات درست کردآیا تبلیغ این ستندویچ فروشی درست بوده؟ فرض بر این است که هر ساندویچ باید نان و حداقل یک نوع گوشت یا پنیر داشته باشد

سلام
ساندویچ فروشه 1000 تا زیاد تر گفته (خالی بسته) .
به 2 طریق می تونه نون و انتخاب کنه . به 3^2 طریق می شه مخلفات و انتخاب کرد (زیر مجموعه ها ی یک مجموعه ی 3 عضوی ). روی هم 7 نوع گوشت و پنیر داریم که 7^2 طریق می شه از اونها انتخاب کرد (زیر مجموعه های یک مجموعه ی 7 عضوی) ولی چون باید حداقل یکی از آنها را انتخاب کنید در نتیجه حالتی که هیچکدام را انتخاب نکنیم حذف می شود . در نتیجه حالت های کل برابر است با :
2*3^2*(1-7^2) = 4^2*(1-7^2) = 4^2-11^2 = 16-2048 = 2032
در نتیجه این ساندویچ فروش 2032 نوع ساندویچ می تواند به مردم بدهد(مگر اینکه بشه ساندویچ دونونه بده که اونوقت 4064 نوع ساندویچ می تونه درست کنه). :happy:

ironroad
02-03-2007, 17:16
سلام
ساندویچ فروشه 1000 تا زیاد تر گفته (خالی بسته) .
به 2 طریق می تونه نون و انتخاب کنه . به 3^2 طریق می شه مخلفات و انتخاب کرد (زیر مجموعه ها ی یک مجموعه ی 3 عضوی ). روی هم 7 نوع گوشت و پنیر داریم که 7^2 طریق می شه از اونها انتخاب کرد (زیر مجموعه های یک مجموعه ی 7 عضوی) ولی چون باید حداقل یکی از آنها را انتخاب کنید در نتیجه حالتی که هیچکدام را انتخاب نکنیم حذف می شود . در نتیجه حالت های کل برابر است با :
2*3^2*(1-7^2) = 4^2*(1-7^2) = 4^2-11^2 = 16-2048 = 2032
در نتیجه این ساندویچ فروش 2032 نوع ساندویچ می تواند به مردم بدهد(مگر اینکه بشه ساندویچ دونونه بده که اونوقت 4064 نوع ساندویچ می تونه درست کنه). :happy:

عالی بود
سپاسگزارم

Moh3en_DDD
03-03-2007, 21:36
پاره خطی به طول رادیکال A بکشید !

ironroad
04-03-2007, 15:32
از ميان N عدد m تاي آنرا به تصادف انتخاب ميكنيم

با توجه به m عدد ظاهر شده آيا مي توان محدوده N را حدس زد

m_honarmand_j
08-03-2007, 18:44
سلام
به چند طریق می شود n نفر را دور یک میز چید؟

سلام
راه حل اول:این افراد را به !n طریق می توان چید که هر حالت n بار ظاهر می شود درنتیجه جواب برابر !(n-1) می شود.
راه حل دوم : فرد 1 را در نظر می گیریم و میز را طوری می چرخانیم که این فرد همیشه در بالا قرار گیرد . در نتیجه سایر افراد را می توان به !(n-1) طریق می توان چید.

m_honarmand_j
08-03-2007, 19:05
سلام
برای آنکه بتوان راس های گرافی را با دو رنگ طوری رنگ کرد که هیچ دو راس همسایه ای رنگ مشترک نداشته باشند حد اکثر چند یال می توان رسم کرد ؟

سلام
برای آنکه بتوان راس های یک گراف را با دورنگ رنگ کرد که هیچ دو همسایه ای رنگ مشترک نداشته باشند باید گراف ما دو بخشی باشد . اگر تعداد راس های گراف زوج باشد آن ها را به دو دسته ی مساوی و اگر تعداد فرد بود آنها را به دودسته که یکی از آن یکی یک راس بیشتر دارد تقسیم می کنیم و در ضمن برای داشتن حداکثر یالها باید گراف دوبخشی کامل باشد . (اگر کسی اثبات اینکه چرا ماکزیمم یال در گراف دوبخشی در حالتی رخ می دهد که دو دسته حداقل تفاضل را داشته باشند را خواست بگوید تا برایش بگذارم)
درجه ی هر راس برابر تعداد راس ها در بخش دیگر است در نتیجه تعداد یالها برابر زیگمای درجه ها تقسیم بر دو است.

m_honarmand_j
08-03-2007, 19:24
سلام
شرط لازم و کافی برای آنکه در یک گراف مسیری وجود داشته باشد که از همه ی یال ها و از هر کدام یک بار بگذرد چیست؟

سلام
حکم معدال این است که بگوییم شرط لازم و کافی برای آنکه در گرافی مسیر اویلری یا مسیر نیمه اویلری وجود داشته باشد چسیت؟
برای آنکه مسیر اویلری وجود داشته باشد باید درجه ی همه ی راس ها زوج و برای آنکه مسیر نیمه اویلری وجود داشته باشد حد اکثر دو راس دارای درجه ی فرد می باشند . (در گراف تعداد راس ها با درجه ی فرد ،زوج است) که این دو راس انتهای مسیر ما می شوند .
اثبات: برای آنکه در مسیر به راسی نرسیم که نتوان از آن خارج شد باید درجه ی راس زوج باشد زیرا تعداد باری که به آن راس وارد می شویم به همان تعداد نیز باید از آن راس خارج شویم در نتیجه باید درجه ی آن زوج باشد . در نتیجه راسی که از آن شروع می کنیم راس پایانی ما نیز می تواند باشد .
به همین ترتیب اگر دو راس درجه ی فرد در گراف وجود داشته باشد باید آن دو راس در انتهای مسیر ما باشند .
(اگر اثباتی که کردم نامفهوم بود بگید تا بیشتر توضیح بدم)

m_honarmand_j
11-03-2007, 14:47
از ميان N عدد m تاي آنرا به تصادف انتخاب ميكنيم

با توجه به m عدد ظاهر شده آيا مي توان محدوده N را حدس زد

سلام
فکر نکنم بدون اینکه در باره ی N و m اطلاعاتی داشته باشیم بتونیم محدوده ی N رو حدس بزنیم .
مثلا اگر اختلاف m و N کم باشه می شه تاحدودی محدوده ی N و حدس زد ولی هرچی اختلاف بیشتر می شه درصد خطا بالا می ره .

m_honarmand_j
11-03-2007, 14:55
سلام
A و B به ترتیب با دومهره ی شاه سیاه و سفید بازی می کنند . هر کس در نوبت خود شاه خود را به یکی از خانه هایی که قبلا توسط یکی از دو شاه اشغال نشده حرکت می دهد . بازنده کسی است که نتواند حرکتی را انجام دهد . در این بازی چه کسی برنده می شود؟

سلام
نفر دوم می برد .
در حالت کلی وقتی به اینجور مسائل بر می خورید که روی یک صفحه بازی ای انجام می شه بیشتر به تقارن مرکزی فکر کنید . اکثر این بازی ها با همین استراتژی حل می شن . تقرریبا می شه گفت که نفر دوم حرکت نفر اول و تقلید می کنه در نتیجه همیشه می تونه حرکت کنه .

m_honarmand_j
11-03-2007, 15:06
سلام
آیا می توان 250 قطعه ی چوبی 1*1*4 را در یک خانه ی 10*10*10 قرار داد؟

سلام
با رنگ آمیزی به وسیله ی چهار رنگ ببینید مسئله حل می شه .

m_honarmand_j
11-03-2007, 15:15
سلام
در یک گراف باید حداقل چند یال داشته باشیم تا بتوانیم یک مثلث در آن پیدا کنیم؟

سلام
اگر تعداد راس های گراف n باشد آنگاه با اصل لانه کبوتری ثابت کنید (البته راه های اثبات دیگری نیز وجود دارد) حداقل این مقدار برابر کچکترین عدد بزرگتر یا مساوی n^2/4 است .

m_honarmand_j
11-03-2007, 15:33
سلام
با استقرا ثابت کنید اگر از بین اعداد 1 تا 2n ، بخواهیم 1+n عدد را انتخاب کنیم ، در بین اعداد انتخاب شده یکی هست که مضرب دیگری است .

m_honarmand_j
11-03-2007, 15:37
سلام
مسئله ی قبل را بدون استفاده از استقرا ثابت کنید .

m_honarmand_j
15-03-2007, 23:39
سلام
بلاخره نتایج مرحله ی اول المپیاد ریاضی رو دادند . ولی هنوز نتایج کامپیوتر رو ندادند .
از استان ما 9 نفر قبول شدند ( البته من هم قبول شدم) که چهار نفرشون از مدرسه ی ماست .
امیدوارم همه ی اونهایی که تلاش می کنن در مرحله ی دوم هم قبول بشوند .

m_honarmand_j
18-03-2007, 11:51
سلام
بلاخره جواب کامپیوتر و هم دادند (البته من در کامپیوتر هم قبول شدم)
دوستان من تقریبا تا 15 فروردین نمی تونم سوالی بزارم .
عید همتون هم مبارک باشه .
فعلا خدانگهدار همتون .

lopez
01-04-2007, 16:20
سلام دوستان
من اثبات اين مسئله رو مي خوام:
تعداد مربع هاي ، صفحه شطرنج چند تا است؟

لطفا اثبات رو برام بذارين

ممنون :40:

mofidy1
01-04-2007, 16:53
سلام
با استقرا ثابت کنید اگر از بین اعداد 1 تا 2n ، بخواهیم 1+n عدد را انتخاب کنیم ، در بین اعداد انتخاب شده یکی هست که مضرب دیگری است .

با سلام

آقای هنرمند ضمن تبریک به مناسبت قبولیتان در مرحله اول المپیاد ریاضی و کامپیوتر، این مساله بدون استفاده از استقراء در لینک زیر حل شده است:

[ برای مشاهده لینک ، لطفا با نام کاربری خود وارد شوید یا ثبت نام کنید ]

ضمنا به عنوان شیرینی قبولیتان، فکر می کنم بهترین کار حل تستهای ترکیبیات المپیاد امسال ریاضی و کامپیوتر در اینجا و حل بقیه تستها در اتاق ریاضیات باشد. اگر لطف کنید و زحمت اینکار را بکشید، ممنون خواهیم شد.

موفق باشید.

12 فروردین 1386

Ar@m
03-04-2007, 22:18
سلام دوستان
من اثبات اين مسئله رو مي خوام:
تعداد مربع هاي ، صفحه شطرنج چند تا است؟

لطفا اثبات رو برام بذارين

ممنون :40:

صفحه شطرنج یک مربعه که اگه هر مربع کوچیک سیاه یا سفید رو یک واحد در نظر بگیریم تعداد مربع های یک واحدی می شه 8*8=64
حالا اگه مربع های دو واحدی رو به همین ترتیب حساب کنیم می شه: 7*7=49
سه واحدی: 6*6=36
چهار واحدی:5*5=25
پنج واحدی: 4*4=16
شش واحدی:3*3=9
هفت واحدی: 2*2=4
و بالاخره هشت واحدی:1*1=1
که چمعا می شه 204 تا

m_honarmand_j
04-04-2007, 13:48
با سلام

آقای هنرمند ضمن تبریک به مناسبت قبولیتان در مرحله اول المپیاد ریاضی و کامپیوتر، این مساله بدون استفاده از استقراء در لینک زیر حل شده است:

[ برای مشاهده لینک ، لطفا با نام کاربری خود وارد شوید یا ثبت نام کنید ]

ضمنا به عنوان شیرینی قبولیتان، فکر می کنم بهترین کار حل تستهای ترکیبیات المپیاد امسال ریاضی و کامپیوتر در اینجا و حل بقیه تستها در اتاق ریاضیات باشد. اگر لطف کنید و زحمت اینکار را بکشید، ممنون خواهیم شد.

موفق باشید.

12 فروردین 1386

سلام
خیلی منمنون از شما . حتما سعی می کنم تا اونجا که بتونم حل سوالات المپیاد رو بزارم و همچنین مسائلی که در اتاق ترکیبیات بدون جواب موندن رو حل شون رو بزارم . :5:

m_honarmand_j
04-04-2007, 14:23
سلام دوستان
من اثبات اين مسئله رو مي خوام:
تعداد مربع هاي ، صفحه شطرنج چند تا است؟

لطفا اثبات رو برام بذارين

ممنون :40:

سلام
برای این مسئله یکی از دوستان جواب گذاشتن ولی حس کردم که باید یک کم بیشتر توضیح داده بشه .
ستون هارا از چپ به راست از 1 تا 8 شماره گذاری کنیم و سطر ها را از بالا به پایین از 1 تا 8 شماره گذاری کنیم .
برای بدست آوردن تعداد مربع های n*n ما مربع های خاص را می شماریم . هر مربع با یک مربع خاص که در گوشه ی سمت چپ-بالا ی آن مربع قرار دارد به طور یکتا مشخص می شود . این مربع ها را مربع خوب n می نامیم .
پس برای بدست آوردن مربع ها کافی است این مربع های خاص را بشماریم .
تعداد مربع های 1*1 برابر با 64 (8*8) می شود .
تعداد مربع های 2*2 برابر با 49 (7*7) می شود زیرا مربع هایی که در سطر 8 و یا در ستون 8 هستند نمیتوانند مربع خوب 2 باشند در نتیجه 7 سطر و ستون می ماند که مربع های آن می توانند مربع خوب 2 باشند .
به همین ترتیبب مربع های خوب 3 تنها مربع های 6 سطر اول و 6 ستون اول می توانند باشند که تعداد آنها 36 می شود . اگر به همین ترتیب پیش برویم تعداد مربع های صفحه ی شطرنج بدست می آید .
اگر در اثبات نکته ای رو بد گفتم بگید تا بیشتر توضیح بدم . :5:
حالا که صحبت صفحه ی شطرنج پیش اومده شما ها تعداد مستطیل های صفحه ی شطرنج رو بدست بیارید .:31:

m_honarmand_j
04-04-2007, 14:34
سلام دوستان
من اثبات اين مسئله رو مي خوام:
تعداد مربع هاي ، صفحه شطرنج چند تا است؟

لطفا اثبات رو برام بذارين

ممنون :40:

سلام
برای این مسئله یکی از دوستان جواب گذاشتن ولی حس کردم که باید یک کم بیشتر توضیح داده بشه .
ستون هارا از چپ به راست از 1 تا 8 شماره گذاری کنیم و سطر ها را از بالا به پایین از 1 تا 8 شماره گذاری کنیم .
برای بدست آوردن تعداد مربع های n*n ما مربع های خاص را می شماریم . هر مربع با یک مربع خاص که در گوشه ی سمت چپ-بالا ی آن مربع قرار دارد به طور یکتا مشخص می شود . این مربع ها را مربع خوب n می نامیم .
پس برای بدست آوردن مربع ها کافی است این مربع های خاص را بشماریم .
تعداد مربع های 1*1 برابر با 64 (8*8) می شود .
تعداد مربع های 2*2 برابر با 49 (7*7) می شود زیرا مربع هایی که در سطر 8 و یا در ستون 8 هستند نمیتوانند مربع خوب 2 باشند در نتیجه 7 سطر و ستون می ماند که مربع های آن می توانند مربع خوب 2 باشند .
به همین ترتیبب مربع های خوب 3 تنها مربع های 6 سطر اول و 6 ستون اول می توانند باشند که تعداد آنها 36 می شود . اگر به همین ترتیب پیش برویم تعداد مربع های صفحه ی شطرنج بدست می آید .
اگر در اثبات نکته ای رو بد گفتم بگید تا بیشتر توضیح بدم . :5:
حالا که صحبت صفحه ی شطرنج پیش اومده شما ها تعداد مستطیل های صفحه ی شطرنج رو بدست بیارید .:31:

abay
05-04-2007, 17:53
سلام
6 نفر در یک مهمانی حضور دارند احتمال اینکه حداقل دو نفر در یک روز هفته متولد شده با شند چیست؟

m_honarmand_j
05-04-2007, 18:14
سلام
با استقرا ثابت کنید اگر از بین اعداد 1 تا 2n ، بخواهیم 1+n عدد را انتخاب کنیم ، در بین اعداد انتخاب شده یکی هست که مضرب دیگری است .

سلام
فرض می کنیم این حکم تا 2n صحیح باشد (پایه ی استقرا رو خودتون برسی کنید)حال ثابت می کنیم اگر از بین اعداد 1 تا 2n+2 بخواهیم n+2 عدد انتخاب کنیم ، دو عدد وجود دارند که بر هم بخشپذیر باشند .
اگر از بین اعداد 2n+1 و 2n+2 یکی انتخاب شود ، مسئله تبدیل به حالتی می شود که از بین اعداد 1 تا 2n می خواهیم n+1 عدد انتخاب کنیم و طبق استقرا دو عدد پیدا می شوند که بر هم بخشپذیر باشند . حال اگر هر دو عدد 2n+1 و 2n+2 انتخاب شوند اگر اعداد 1 یا 2 یا n+1 انتخاب شوند ، یکی بر دیگری بخشپذیر می شود . حال اگر بخواهیم مانند قبل عمل کنیم باید از بین اعداد 1 تا 2n تعداد n تا عدد انتخاب کنیم و نمی توانیم از فرض استقرا استفاده کنیم . نکته ی این سوال هیمن جا است . ما به n عدد انتخاب شده عدد n+1 را اضافه می کنیم و ثابت می کنیم اضافه کردن این عدد به مسئله لطمه وارد نمی کند . اگر این عدد را اضافه کنیم اولین مضرب آن 2n+2 است پس به n عدد ما لطمه نمیزند . فقط می ماند ممکن است که یکی از n عدد انتخاب شده مقسوم علیه n+1 باشد . اگر این چنین شد آن عدد مقسوم علیه 2n+2 نیز هست . پس اضافه کردن این عدد مشکلی را ایجاد نمی کند و ما از بین اعداد 1 تا 2n تعداد n+1 عدد انتخاب کرده ایم و طبق فرض استقرا دو عدد پیدا می شوند که یکی بر دیگری بخشپذیر باشد . :21:

m_honarmand_j
05-04-2007, 18:24
سلام
یک مسئله ی بقول معروف گفتنی پیکار جو :
یک صفحه ی شطرنجی نامتناهی وجود دارد که در آن تعداد متناهی خانه را سیاه کرده ایم به طوری که هر خانه ی سیاه با زوج تا خانه ی دیگر سیاه همسایه است . هر دو خانه ای که یک ضلع مشترک دارند همسایه هستند. ثابت کنیم می توان خانه های سفید را با دو رنگ آبی و قرمز طوری رنگ کرد که هر خانه ای سیاهی که در نظر بگیریم تعداد همسایه های آبی اش با تعداد همسایه های قرمز اش برابر باشد .

m_honarmand_j
07-04-2007, 01:50
سلام
این فعلا سوالات مرحله ی اول کامپیوتر امسال تا جواباشون و هم بزارم .
[ برای مشاهده لینک ، لطفا با نام کاربری خود وارد شوید یا ثبت نام کنید ]

m_honarmand_j
07-04-2007, 14:58
سلام
این سوالی که می خوام بزارم و یکی از دوستان در قسمتی دیگر (سوال جبر و احتمال)گذاشته بودند و جواب خواسته بودند. من این سوال و این جا می زارم ، اگر کسی جوابی نداد بعد از مدتی خودم جوابشو می گم :
در یک مسابقه ی ریاضی 6 سوال به شرکت کنندگان داده شده است . هر جفت از سوالات به وسیله ی بیش از 5/2 از شرکت کنندگان حل شده است . هیچ کس هم تمام 6 سوال را حل نکرده است . ثابت کنید حداقل دو نفر هستند که دقیقا 5 سوال از 6 سوال را حل کرده باشند .

ali_hp
08-04-2007, 00:31
سلام
منم هردو تاشو نو تبریک می گم (مخصوصا کامپیوتر!) ایشالا مراحل بعدی و ...
الان اون سوال معبد شایولین دیدم اخرشم یه شرط مساله رو نگفتی غیر از یک نفرکه دورغ گو است می دانیم چهار نفر دیگر راستگو هستند.
اینم یه سوال:
n نقطه متمایز در صفحه در نظر بگیرید همه پاره خطهایی که از اتصال دو به دوی این نقاط به دست می ایند را در نظر می گیریم و وسط هریک از این پاره خطها را به رنگ قرمز در می اوریم حد اقل تعداد نقاط قرمز بدست امده چند تاست؟

m_honarmand_j
08-04-2007, 18:29
سلام
سوال اول که در لینک آمده به دلیلی که جوابش تو گزینه ها نبود حذف شد .
[ برای مشاهده لینک ، لطفا با نام کاربری خود وارد شوید یا ثبت نام کنید ]

m_honarmand_j
08-04-2007, 18:32
سلام
سوال دوم:([ برای مشاهده لینک ، لطفا با نام کاربری خود وارد شوید یا ثبت نام کنید ])
هر سال، هر نهنگی که متوجه شود هیچ نهنگ زنده ای را دوست ندارد، خودکشی می کند. شکل زیر یک جامعه از نهنگ ها را نشان می دهد. اگر نهنگ A به B پیکان داشته باشد، یعنی A، B را دوست دارد. چند سال طول می کشد تا همه ی این نهنگ ها خودکشی کنند؟

الف) 8 ب) 9 ج) 10 د) 11 ﻫ) هیچ کدام
پاسخ: هیچ کدام . اگر در شکل دقت کنید کی بینید که یک دور شامل سه راس ایجاد می شود در نتیجه این سه نهنگ همیشه حد اقل یک دوست زنده دارند .

m_honarmand_j
21-04-2007, 19:37
سلام
) نمایندگان شرکت های تولید کننده ی گوشت، مورد بازجویی قرار گرفتند. اظهارات آنان بدین شرح است:

* شرکت A: شرکت B گوشت فاسد به مردم می دهد.

* شرکت B: شرکت A گوشت فاسد به مردم می دهد.

* شرکت C: شرکت های A و B، هر دو گوشت فاسد به مردم می دهد.

* شرکت D: از بین شرکت ما و شرکت A حداقل یکی گوشت فاسد به مردم می دهد.

نماینده ی هر شرکتی که از گوشت فاسد استفاده می کند، از آنجا که آدم بدی است، همواره دروغ می گوید. همچنین، نماینده ی هر شرکتی که گوشت سالم به مردم می دهد، همواره راست می گوید. چند شرکت از گوشت فاسد استفاده می کنند؟

الف) 1 ب) 2 ج) 3 د) 4

ﻫ) نمی توان با اطمینان پاسخ گفت.

جواب: اگر کمی منطقی فکر کنید و به سوال خوب توجه کنید می بینید که دو شرکت گوشت فاسد به مردم می دهند.

m_honarmand_j
21-04-2007, 19:42
سلام
[ برای مشاهده لینک ، لطفا با نام کاربری خود وارد شوید یا ثبت نام کنید ]
سوال4:) می خواهیم در یک جعبه ی مکعبی ، یک جسم سه بُعدی نامعلوم را قرار بدهیم که:

* دقیقا از تعداکد صحیحی مکعب تشکیل شده باشد.

* یک تکه باشد؛ اگر یک نقطه از آن را گرفته و جسم را به سمت بالا بکشیم، به طور کامل بالا بیاید. فرض کنید دو مکعب کوچک () حتی اگر در یک نقطه (مثلا گوشه) با هم تماس داشته باشند، آن گاه به هم چسبیده اند.

* اگر در جعبه را ببندیم و جعبه را تکان بدهیم، جسم در داخل آن ثابت بماند. می توانید فرض کنید اتّصالات جسم (حتی در گوشه ها) بسیار محکم هستند!

حداقل حجم چنین جسمی چند واحد مکعب است؟

الف) 3n-2 ب) 3n-3 ج) 2n د) 2n-1 ﻫ) n

جواب: گزینه ی ه صحیح است و شکل ما شامل n مکعب 1*1*1 است که از یک گوشه ی شکل به گوشه ی مقابل آن(به صورت قطری)کشیده شده است و واضح است که تعداد مکعب های 1*1*1 آن n تاست.

m_honarmand_j
22-04-2007, 18:44
سلام
سوال 5: 5) دو نفر این بازی را انجام می دهند: آنها در یک جدول (یک سطر با n خانه) به نوبت مهره می گذارند. هر خانه ی جدول گنجایش یک مهره را دارد و در ابتدا جدول خالی است. نفر اول در هر حرکت یک مهره ی سفید و نفر دوم در هر حرکت یک مهره ی سیاه در یک خانه ی خالی می گذارند. وقتی مهره ای - مثل A - در جدول قرار می گیرد، اگر مهره ای همرنگ A - مثل B - در جدول باشد و تمام خانه های بین A و B با مهره هایی با رنگی مخالف ِ رنگ ِ A پر شده باشند، تمام مهره های بین A و B، همرنگ ِ A می شوند. با اضافه کردن A به جدول ممکن است دو مهره مانند B در جدول (در دو طرف A) پیدا شوند که شرایط گفته شده را داشته باشند، که در این صورت عمل مذکور را در هر مورد انجام می دهیم.


بعد از n حرکت، تمام خانه های جدول پر می شوند و بازی تمام می شود. در این زمان اگر تعداد مهره های سفید درون جدول بیشتر باشد، نفر اول می برد، اگر تعداد مهره های سیاه بیشتر باشد، نفر دوم می برد، وگرنه مساوی می شوند. برای n های بزرگ تر از 10، کدام جملات زیر درست اند؟

1) برای n های فرد نفر اول راهکار برد دارد.

2) برای n های فرد نفر دوم راهکار برد دارد.

3) برای n های زوج نفر اول راهکار برد دارد.

4) برای n های زوج نفر دوم راهکار برد دارد.

5) برای n های زوج نفر اول راهکار نباختن دارد.

6) برای n های زوج نفر دوم راهکار نباختن دارد.

الف) 1 و 3 و 5 ب) 2 و 4 و 6 ج) 1 و 4 و 6

د) 2 و 3 و 5 ﻫ) 1 و 5 و 6

جواب: استراتژی برد برای نفر اول شروع کردن از یک طرف و گرفتن هردو طرف است و یعنی باید در دوگوشه مهره ی خود را بگذارد و نفر دوم نیز باید وقتی که نفر اول مهره ی خود را در گوشه ای گذاشت ، مهره اش را در گوشه ی مقابل بگذارد . در حرکت های بعد باید به ترتیب از همان طرفی که شروع کرده اند ادامه دهند و مهره های خود را بگذارند . به طور خلاصه هرکس باید ازیک گوشه ی صفحه شروع کند و مهره هایش را بگذارد . می توانید برسی کنید که اگر کسی حرکتی دیگر انجام دهد ، نفر دیگر می تواند طوری بازی کند که ببرد .
به سادگی نتیجه می شود وقتی که n زوج باشد نفر اول می تواند n/2 و وقتی که n فرد باشد سقف n/2 را بگیرد در نتیجه گزینه ی ه صحیح است .

m_honarmand_j
22-04-2007, 19:06
سلام
سوال6 : ) 1385 نفر روی محور اعداد صحیح ایستاده اند، به طوری که نفر i ام () روی نقطه به طول i قرار گرفته است. هر کدام از این افراد یک سنگ دارد و در یک لحظه همه با هم سنگ شان را به طرف مثبت پرتاب می کند. بُرد ِ سنگ i ا ُم را با f(i) نشان می دهیم. بدین ترتیب، سنگ نفر i ام بعد از پرتاب در محل i+f(i) قرار می گیرد. می دانیم به ازای هر x، f(x) برابر است با تعداد «صفر» ها در نمایش مبنای دوی x (سمت چپ ترین رقم در نمایش مبنای دو همواره یک است). به عنوان مثال، f(13) برابر است با 1، زیرا نمایش مبنای دوی 13 به صورت «1101» می باشد که سه عدد «1» ویک عدد «0» دارد.


بعد از این که همه سنگ خود را پرتاب کردند، دورترین سنگ کجا می افتد؟ (بزرگ ترین مکانی که در آن حداقل یک سنگ قرار می گیرد کجاست؟)

الف) 1385 ب) 1386 ج) 1390 د) 1391 ﻫ) 1395

جواب: گزینه ی ج جواب درست است . به سادگی می توان دید که به مکان بزرگتری نمی توان دست یافت .
این سنگی که در این مکان قرار می گیرد به وسیله ی کسی پرتاب شده است که در خانه ی شماره ی 1385 قرار دارد . کافی است که نمایش این عدد در مبنای دو را در نظر بگیرید . اگر بخواهیم که تعداد صفر ها را از این بیشتر کنیم نمیتوانیم عدد را بزرگتر کنیم و اگر این عدد را بخواهیم کوچکتر کنیم به ازای هر صفر(بجز صفری که در سمت راست قرار دارد که با ازای آن یک واد از عدد کم می شود و یک واحد به f آن عدد اضافه می شود) باید حد اقل چهار واحد از عدد کم کنیم تا بتوانیم صفری دیگر را اضافه کنیم ولی در مجموع حد اقل 3 واحد از جایی که سنگ نفر 1385 قرار دارد به عقب باید برویم .

m_honarmand_j
22-04-2007, 19:11
سلام
[ برای مشاهده لینک ، لطفا با نام کاربری خود وارد شوید یا ثبت نام کنید ]
سوال7) یک میمون در نقطه ی (0,0) صفحه ی مختصات قرار دارد. این میمون در هر حرکت می تواند نقطه ای با مختصات صحیح را انتخاب کند که از نقطه ی فعلی اش،‌ فاصله ای بیشتر از 10 واحد نداشته باشد. سپس، حول آن 90 درجه دوران یافته و در نقطه ی جدید قرار بگیرد. نمونه ای از حرکت های میمون را در شکل مشاهده می کنید.



این میمون به چند تا از نقطه های زیر می تواند برسد؟

(5,13) , (13,21) , (21,33)

(1385,2007) , (1024,2048) , (55,255)

الف) 1 ب) 2 ج) 4 د) 5 ﻫ) 6

جواب : گزینه ی ه صحیح است .(نمی دونم چرا . من خودم این سوال و سر جلسه حل نکردم و بعدا هم روش فکر نکردم)

m_honarmand_j
22-04-2007, 20:25
سلام
دوستان ببخشید که در یک مدت طولانی هیچ پستی نداشتم و جوابی به سوالات شما ندادم . واقعا سرم شلوغه چون این هفته مرحله ی دوم ریاضی و دو هفته بعد مرحله ی دوم کامپیوتره . اینشا الله بعد از مرحله ی دوم کامپیوتر وقت آزاد بیشتری دارم . شما هاهم دعا کنید که من قبول بشم .

B O L O T
22-04-2007, 21:01
سلام
دوستان ببخشید که در یک مدت طولانی هیچ پستی نداشتم و جوابی به سوالات شما ندادم . واقعا سرم شلوغه چون این هفته مرحله ی دوم ریاضی و دو هفته بعد مرحله ی دوم کامپیوتره . اینشا الله بعد از مرحله ی دوم کامپیوتر وقت آزاد بیشتری دارم . شما هاهم دعا کنید که من قبول بشم .

ممنون ميشم ادامه بديد

m_honarmand_j
23-04-2007, 15:36
ممنون ميشم ادامه بديد

سلام
به روی چشم دوست عزیز . حتما ادامه خواهم داد .

m_honarmand_j
23-04-2007, 15:57
سلام
[ برای مشاهده لینک ، لطفا با نام کاربری خود وارد شوید یا ثبت نام کنید ]
8) یک جدول 5*5 داریم، که در هر خانه ی آن یک تخم مرغ قرار دارد. می خواهیم تعدادی از این تخم مرغ ها را برداریم. در هر خانه که قرار داشته باشیم یا از آن رد شویم، می توانیم تخم مرغ آن خانه را برداریم. در ابتدا از یکی از خانه های جدول شروع به حرکت می کنیم. در هر مرحله دقیقا یک حرکت انجام می دهیم و در هر حرکت به یکی از خانه های مجاور (دارای ضلع مشترک با خانه ی فعلی) می رویم. اگر در مرحله ای جهت حرکت ما تغییر کند (از افقی به عمودی یا از عمودی به افقی)، باید 1 تومان جریمه بدهیم. اگر ما 5 تومان پول داشته باشیم، حداکثر چند تخم مرغ می توانیم برداریم؟


الف) 20 ب) 21 ج) 22 د) 24 ﻫ) 25

جواب:گزینه ی ب جواب صحیح است . باید از یک گوشه شروع کنیم و به صورت مارپیچ حرکت کنیم(در امتداد اضلاع) . یعنی مستقیم حرکت کنیم و تا انتها برویم سپس بپیچیم و مستقیم برویم و دوباره هنگامی که به انتها (تا جایی که در آن تخم مرغ باشد ) رسیدیم بپیچیم . مشاهده می شود که با این کار با 5 بار تغییر مسیر می توان 21 تخم مرغ را برداشت و همچنین می توان برسی کرد که امکان برداشتن تخم مرغ بیشتر نیز نمی باشد . (واقعا نمی دونم چه جوری باید این مسائل و اثبات کرد . یه جورایی این سوال ها حسی ان . شاید هم بشه اثبات دقیقی پیدا کرد ولی گفتنش یکم طولانی می شه)

m_honarmand_j
23-04-2007, 16:05
سلام
[ برای مشاهده لینک ، لطفا با نام کاربری خود وارد شوید یا ثبت نام کنید ]
سوال9) 12 نهنگ که قصد خودکشی دارند، در یک صف قرار گرفته اند. یک روز صبح نهنگ ها تصمیم گرفتند که از آن روز به بعد، صبح هر روز، اگر نهنگ زنده ای در صف وجود داشته باشد، تعدادی (ناصفر) از این نهنگ ها خودکشی کنند. در صورتی که بعد از خودکشی صبح یک روز، هنوز نهنگ زنده ای در صف وجود داشت، همان شب هم تعدادی (ناصفر) خودکشی می کنند. واضح است که نهنگ ها به همان ترتیبی که در صف ایستاده اند خودکشی می کنند. این 12 نهنگ به چند طریق می توانند خودکشی کنند، به طوری که در پایان تعداد نهنگ هایی که در صبح خودکشی کرده اند،‌ با تعداد نهنگ هایی که در شب خودکشی کرده اند برابر باشد؟

الف) 720 ب) 924 ج) 462 د) 360 ﻫ) 1440

جواب : گزینه ی صحیح ج است .
روش اول : ما باید 6 تا از نهنگ ها را انتخاب کنیم که این کار به انتخاب 6 از 12 (924) طریق امکان دارد ولی هر مجموعه را دوبار می شماریم زیرا مثلا فرض کنید 6 نهنگ اول انتخاب شوند و بار دیگر 6 نهنگ دوم ولی باید به این نکته توجه کرد که نهنگ اول باید در دسته ای قرار گیرد که در روز خودکشی می کنند . در نتیجه به 462 طریق می توان این کار را کرد .
روش دوم : نهنگ اول باید در روز خود کشی کند در نتیجه از 11 نهنگ باقی مانده باید 5 تا را انتخاب کنیم تا در روز خود کشی کنند که این کار به 462 طریق انجام می شود(انتخاب 5 از 11).

m_honarmand_j
23-04-2007, 16:18
سلام
یک جدول با دو سطر و n ستون داریم . در خانه های این جدول دست کم دو به توان n مهره وجود دارد . در هر حرکت می توان از یک خانه که بیش از 1 مهره در آن باشد دو مهره برداشت و یک مهره در خانه ی بالایی یا سمت راستی اش قرار دهیم . ثابت کنید میتوان مهره هارا طوری جابجا کرد که یک مهره به خانه ی راست-بالا برسد . (مرحله ی دوم بیست و سومین المپیاد ریاضی ایران )

m_honarmand_j
24-04-2007, 20:13
سلام
امروز نوبت اول مرحله ی دوم ریاضی برگزار شد و فردا نوبت دوم آن برگزار می شود .
فعلا تا فردا .

m_honarmand_j
05-05-2007, 01:35
سلام
[ برای مشاهده لینک ، لطفا با نام کاربری خود وارد شوید یا ثبت نام کنید ]


10) در شکل زیر(برای دیدن شکل به لینک داده شده مراجعه کنید) هر کدام از مهره های داخل صفحه یک مهره ی ماهی است. مهره ی ماهی در صفحه سُر می خورد. یعنی در یکی از 4 جهت حرکت می کند تا به یک خانه ی پُر برسد و در خانه ی خالی ِ قبل از خانه ی پُر متوقّف می شود. قوانین بازی به شرح زیر است:



* سفید برنده است اگر یک مهره اش را به سطر آخر (پایین) برساند.

* سیاه برنده است اگر یک مهره اش را به سطر اول (بالا) برساند.

* خانه های سیاه و خانه هایی که مهره ای در آن ها هست، پر هستند.

* هر کس که نوبتش است، باید یکی از مهره هایش را جا به جا کند.

* کسی حق ندارد عکس حرکت قبلش را انجام دهد.

* سفید اول بازی می کند.

اگر هر دو نفر به بهترین نحو بازی کنند، کدام یک از گزاره های زیر درست است؟

الف) سفید،‌با انجام حداکثر 4 حرکت می برد.
ب) سیاه، می تواند طوری بازی کند که سفید نتواند در چهار حرکت ببرد ولی سفید با حداکثر شش
حرکت می برد.
ج) سیاه،‌با انجام حداکثر 4 حرکت می برد.
د) سفید، می تواند طوری بازی کند که سیاه نتواند در چهار حرکت ببرد ولی سیاه با حداکثر شش
حرکت می برد.
ﻫ) هیچ کدام

جواب: گزینه ی ه یعنی هیچکدام است . (خیلی نامردی ای که جواب سوالی هیچ کدام باشه چون آدم شک می کنه که جواب درسته یا نه) با یک کم بازی کردن می تونید به جای نفر دوم کاری کنید که بازی طول بکشه . اگه یک کم بازی کنید می بینید که بهترین حرکت برای نفر اول سردادن ماهی سمت راست به پایین است و برای نفر دوم هم سردادن ماهی سمت چپ به بالا است . سپس نفر اول ماهی دوم از سمت راست را به پایین سر می دهد و نفر دوم نیز دوباره عکس حرکت او را انجام می دهد و بعد از آن نفر اول ماهی سوم را به پایین سر می دهد . در این هنگام نفر دوم ماهی دوم را که به بالا سر داده بود به سمت چپ(سمت ماهی اولش) سر می دهد . تا اینجا سه حرکت انجام شده است . اگر بتوانید خوب بازی کنید می توانید زوری بازی کنید که حد اقل در شش حرکت هیچ بازیکنی نتواند ببرد .

m_honarmand_j
05-05-2007, 19:17
سلام
11) ربع اول صفحه ی مختصات را مطابق شکل(برای دیدن شکل به لینکی که در پست قبلی وجود دارد بروید) به خانه های تقسیم می کنیم.

به چند طریق می توان اعداد 1 تا 14 را در 14 تا از این خانه ها نوشت به طوری که شرایط زیر برقرار باشند:

* در خانه ی (1,1) (پایین ترین و سمت چپ ترین خانه) عدد 1 نوشته شده باشد.

* اگر در خانه ای عددی نوشته شده بود، دقیقا در یکی از دو خانه ی پایینی یا سمت چپی آن خانه، عددی نوشته شده باشد.

* اگر در خانه ی (x,y) یعنی خانه ی سطر x اُم و ستون y اُم، عدد 1<i نوشته شده بود، هر یک از اعداد 1 تا i-1، در خانه ای مثل (x',y') قرار داشته باشند که و .


الف) 8192 ب) 16384 ج) 24575 د) 24576 ﻫ) هیچ کدام

جواب : گزینه ی الف صحیح است .
اگر بخواهیم در شکل نشان دهیم باید آن را مانند دستگاه مختصات در نظر بگیریم و خانه های (1و14) و (14و1) را رنگ و تمامی خانه های روی خطی که آن ها را به هم وصل می کند را نیز رنگ کنیم . در نتیجه سوال به این تبدیل می شود که به چند طریق می توان به خانه های رنگی با حرکت های رو به بالا و رو به راست (با چهارده حرکت)رفت . در نتیجه جواب برابر است با مجموع انتخاب i از 14 که i بین 1و 14 (و مساوی آن ها ) می باشد . اگر حاصل این عبارت را به دست بیاوریم به گزینه ی الف می رسیم .

m_honarmand_j
09-05-2007, 15:29
سلام بر دوستان
امروز نوبت دوم مرحله ی دوم برگزار شد . خوشبختانه همه ی 8 سوال و جواب دادم . روز اول که تو نصف وقت تموم کردم . اگر خدا بخواهد حتما قبول می شم. برای همه ی کسانی که در این امتحان شرکت کردند آرزوی موفقیت دارم .

m_honarmand_j
11-05-2007, 11:47
سلام
دوستان به زودی سوالات مرحله ی دوم کامپیوتر امسال و با جواباشون براتون می زارم(اگه بشه ریاضی ها رو هم می زارم).

m_honarmand_j
11-05-2007, 11:55
سلام
[ برای مشاهده لینک ، لطفا با نام کاربری خود وارد شوید یا ثبت نام کنید ]
12) در شکل زیر، 15 دفتر اداری یک سازمان نشان داده شده است.(برای دیدن شکل به لینک مراجعه کنید.

هر دفتر، تعدادی دفتر زیردست دارد. این ارتباط در شکل با پیکان هایی از دفتر ها به دفتر های زیردستشان نمایش داده شده است. هر دفتر، بدین صورت عمل می کند: هر نامه ای را که دریافت می کند، برای هر کدام از دفتر های زیردستش کپی می کند و می فرستد (دفتر های زیردست هم همین کار را انجام می دهند). دفتر های ردیف بالا از چپ به راست به ترتیب شماره های 0 تا 4 را دارند. طی 2007 روز، این دفتر ها به این صورت کار کرده اند که در روز i ام () از بیرون سازمان به دفتر k ام ردیف بالا ()، i+k نامه می رسد.


اگر پس از انجام همه ی این عملیات اداری طی این 2007 روز، مجموع تعداد نامه های دریافت شده توسط اداره های سطر پایین، n باشد، باقی مانده ی تقسیم عدد n بر 5 کدام است؟

الف) 0 ب) 1 ج) 2 د) 3 ﻫ) 4

جواب:گزینه ی د صحیح است . (واقعا نمی دونم این سوال و چجوری باید حل کرد . من خودم حلش نکردم . اگه کسی فهمید چجوری ه حلش ازش ممنون می شم هم واسه ی من و هم واسه ی دیگران جوابشو بزاریه)

messengerman2007
14-05-2007, 21:01
دوستان می خواستم بدونم برای محاسبه مجموع اعداد یک جدول ضرب n*n ( ان در ان ) فرمول خاصی هست اگه نیست چه جوری می تونم از آنالیز ترکیبی حلش کنم.

Vmusic
14-05-2007, 21:11
عنوان ویرایش شد !!

در انتخاب عنوان دقت کنید !!

ممنون موفق باشید !!

Iron
15-05-2007, 02:27
سلام علیکم

[ برای مشاهده لینک ، لطفا با نام کاربری خود وارد شوید یا ثبت نام کنید ] ([ برای مشاهده لینک ، لطفا با نام کاربری خود وارد شوید یا ثبت نام کنید ])

m_honarmand_j
18-05-2007, 15:21
سلام
بلا خره یک هفته ی زجر آور تموم شد . تو این هفته 12 تا امتحان دادیم که همشون از کل کتاب بودن . امروز هم امتحان ریاضی رو به بدترین شکل ممکن دادم . بگذریم . سوال اول روز اول الکپیاد کامپیوتر امسال :
هابیل و قابیل دارند بازی می کنند . آن ها در ابتدا n نقطه در صفحه گذاشته اند آن ها را با n خط راست یا منحنی طوریبه هم وصل کرده اند که یک منحنی بسته بوجود آمده که خودش را قطع نکرده است . حال به نوبت بازی می کنند . هر کس در نوبت خود یا می تواند یک ناحیه (بسته یا باز) بدون رنگ را رنگ کند یا دو نقطه را به هم وصل کند که غبلا به هم وصل نبوده اند و این خط جدید هیچ نه از ناحیه ای رنگی بگذرد و نه خط های قبلی را قطع کند . اگر کسی نتواند حرکتی انجام دهد می بازد .
حال اگر هابیل بازی را شروع کند به ازای چه n هایی قابیل می تواند طوری بازی کند که حتما ببرد ؟
جواب: به ازای تمام n ها . راهکار برد قابیل این است که در هر مرحله یک ناحیه ی بدون رنگ را رنگ کند . در اول دوناحیه ی بدون رنگ وجود دارد ، پس هابیل نمی تواند هیچ کدام از آن ها را رنگ کند پس مجبور است یک خط جدید بکشد . این خط جدید دقیقا یک ناحیه به مجموع ناحیه ها اضافه می کند(می توان با فرمول اویلر در مورد گراف های مسطح اثبات کرد) و نفر دوم یک ناحه ی بدون رنگ را رنگ می کند و تعداد ناحیه های بدون رنگ را به 2 تا کاهش می دهد و باز هم مانند حالت اول می شود . به همین ترتیب که پیش برویم بلاخره به جایی می رسیم که نفر اول دیگر نمی تواند خطی جدید بکشد و مجبور است ناحیه های بدون رنگ را به یکی کاهش دهد و نفر دوم با رنگ کردن ناحیه ی دیگر می برد .

m_honarmand_j
18-05-2007, 15:41
سوال دوم : چنگیز خان یه سهری مهمون دعوت کرده ولی نمی خواد بزاره همشون برگردن به شهرشون(همه از یه جا اومدن)دو کشور اونا یه سری شرکت هوایی وجود داره که بین هر دوشهر توسط یک شرکت اداره می شه (ممکن ه یه شرکت چند تا از شهر ها را دو به دو به هم وصل کنه ولی بین هر دوشهر اگه خط هوایی وجود داشته باشه فقت توسط یک شرکت اداره می شه) چنگیز خان از هر شرکت یک بلیط می خره . برای اونکه بشه از شهری به شهره ریگه ای رفت باید بلیط شرکت اداره کننده ی اون خط و داشت . و از هر شهری به هر شهر دیگر می توان رفت (مستقیم یا با واسطه) . چنگیز خان به دوستاش می گه با این بلیط ها همه شون نمی تونن برگردن . اگر هر جفت از بلیط ها رو بگیری حداقل یک نفر می تونه برگرده شهرش .
آیا چنگیز خان دروغ می گه یا می تونه راست بگه؟
جواب : می تونه راست بگه . فقط باید یک شکل کشید و گفت تو این شکل راست می گه(سخت نیست)

m_honarmand_j
18-05-2007, 15:42
دوستان ادامه ی سوالات رو هم می زارم . ولی الان وقت ندارم .

Ar@m
19-05-2007, 23:22
اول برات آرزوی موفقیت در المپیاد می کنم
دوم فکر نمی کردم نمونه سوال المپیاد کامپیوتر اینجوری باشه
اگه ممکنه چند تا دیگه هم بذار لطفا

kaakaa
22-05-2007, 01:35
bego kodomaro hal nakarD vasat hallesh konam.:5:

m_honarmand_j
22-05-2007, 19:51
سوال سوم روز اول المپیاد کامپیوتر امسال: یک تعدادی کلید سالم و خراب در یک ردیف از چپ به راست داریم که یک روبات می خواد اونا رو پیدا کنه . هر کلید یک سیم خروجی داره که کلید سالم اگر بالا باشد سیمش برق دارد و کلید خراب همیشه سیمش برق دار . به روبات می تونیم این دستورات رو بدیم . حالت کلید رو بررسی کن . حالت کلید رو عوض کن . سیم روبه رو را بررسی کن که برق دارد یا نه .برو به کلید بعد یا قبلی . کار خود را به پایان برسان و کلید های خراب را گزارش بده .
ولی روبات ما یک مشکل داره که اگه سیمی که برق دارد رو کنترل کنه مشکل پیدا می کنه و ار کلید اول ادامه ی دستورات رو اجرا می کنه . الگوریتمی اراته دهید که با انجام آن بتواند کلید های خراب را پیدا کند.(فرض کنید که در ابتدا همه ی کلید ها پایین اند)
جواب : کلید مقابل را برسی کن . اگر پایین بود حالت آن را عوض کن: ( سیم مقابل را برسی کن . اگر برق نداشت آن را به پایین بزن . برو به کلید بعدی.) اگر بالا بود ( برو به کلید بعدی) . اگر کلید دیگری نبود کار را متوقف کن و کلید های خراب را گزارش بده .
با انجام دستورات بالا متوجه می شویم که در پایان کلید های خراب در حالت بالا می مانند .

m_honarmand_j
22-05-2007, 19:55
سوال چهارم روز اول مرحله ی دوم المپیاد کامپیوتر امسال : می خواهیم بین n شهر جاده بکشیم . فاصله ی دو شهر را کوتاه ترین مسیر بین آن ها در نظر می گیریم که به ازای فاصله ی هر دوشهر باید آن مقداد عوارض بدهیم . ( به ازای هر یال در مسیر بین آن رو شهر یک واحد) . همچنین احداث هر جاده مخارجی دارد . هزینه ی ساخت یک نقشه ی جاده ها برابر مجموع مخارج جاده ها به همراه مجموع عوارض هر دو شهر می باشد . ( نقشه باید همبند باشد) . کمترین هزینه ی ساخ را بیابید ( در دو حالت برسی کنید . اگر هزینه ی ساخت هر جاده کمتر مساوی 1 باشد و در حالت بعد بیشتر از 1 باشد).
جواب : اگر هزینه ی هر جاده کمتر مساوی 1 باشد باید همه ی جاده ها باشند . یعنی به ازای هر دوشهر یک جاده بینشان باشد .(می توانید برسی کنید که اگر فاصله ی دو شهر بیشتر از 1 باشد هزینه ی ساخت بیشتر می شود) در نتیجه هینه ی ساخت نقشه برابر انخاب 2 از n ( مجموع عوارض ها) بعلاوه ی انتخاب 2 از n ضرب در هزینه ی ساخت یک جاده ( هزینه ی جاده ها) . در نتیجه حاصل برابر است با:
( a هزینه ی ساخت یک جاده) n*(n-1)/2 *( a+1)
در حالت دوم گراف ما باید گراف ستاره ای باشد . یعنی یک شهر در مرکز و دیگر شهر ها تنها به این شهر متصل باشد . در نتیجه هزینه ی ساخت نقشه برابر است با :
a*(n-1) + (n-1)+ 2*((n-1)*(n-2)/2)
(زیرا n-1 جاده داریم و فاصله هر شهر از شهر مرکزی 1 است که n-1 از این فاصله ها است و به تعداد انتخاب 2 از (n-1) تا فاصله ی 2 داریم . )

m_honarmand_j
22-05-2007, 19:58
سوال اول روز دوم مرحله ی دوم المپیاد کامپیوتر امسال : فاصله ی دو رشته به طول n را برابر تعداد مکان هایی می گیریم که با هم در آن جا تفاوت دارند . 1386 رشته به طول n داریم که مجموعه ی A را تشکیل می دهند . یک رشته مانند # به طوب n را در نطر می گیریم که در بین همه ی رشته های به طول n کمترین فاصله را از A دارد . ( مجموع فاصله ی # از تک تک اعضای A ) . این مقدار فاصله را a در نظر می گیریم . ثابت کنید که یک رشته به طول n در مجوعه ی A وجود دارد که فاصله اش از A حداکثر 2a است .
جواب : برای این کار باید ابتدا ثابت کنیم که اگر دو رشته از رشته ای دیگر فاصله های x و y را دارا باشند آنگاه این دو رشته از هم فاصله ای حداکثر x+y دارند . برای این کار اگر خلاف آن را فرض کنیم به تناقض می رسیم . بعد رشته ای در مجموعه ی A را در نظر می گیریم که کمترین فاصله با # را دارد . ثابت می کنیم که این رشته از A حداکثر فاصله ای به اندازه ی 2a را دارد . برای این کار می توان بر روی تعداد رشته ها استقرا زد یا روشی دیگر . ولی به سادگی این قسمت اثبات می شود . در این مسئله مهم این بود که باید می فهمیدیم که اگر دورشته از رشته ای فاصله های x و y را داشته باشند از هم دیگر حداکثر فاصله ی x+y را دارند .

m_honarmand_j
22-05-2007, 19:59
سوال دوم روز دوم مرحله ی دوم المپیاد کامپیوتر امسال : سه مجموعه A و B و C وجود دارند . مجموعه ی A+B+C اینگونه ساخته می شود که d عضو این مجموعه است اگر و فقط اگر d=a+b+c باشد که a عضو A و b عضو B و c عضو C است . اگر تعداد اعضای سه مجموعه ی اولیه به ترتیب m و n و k باشد ، حداقل مجموعه ی A+B+C چند عضو دارد؟
جواب : m+n+k-2 تا .
اگر اعضای هر مجموعه را یصورت صعودی مرتب کنیم داریم :
a1+b1+c1<a2+b1+c1<…<am+b1+c1<am+b2+c1<am+b3+c1<…<am+bn+c1<am+an+c2<am+bn+c3<…<am+bn+ck .
که تعداد آن ها m+n+k-2 تا است . مثلا فرض کنیم m<=n<=k و اعضای مجموعه ی A از x شروع و به x+m-1 ختم شوند و مجموعه ی B از x شروع و به x+n-1 ختم شود و C از x شروع و به x+k-1 ختم شود آنگاه مجموعه ی A+B+C از 3x شروع و به 3x+m+n+k-3 ختم می شود که در نتیجه تعداد آنها m+n+k-2 تا است .

m_honarmand_j
22-05-2007, 20:06
سوال سوم روز اول مرحله ی دوم المپیاد کامپیوتر امسال : 2n نفر به مسافرتی رفته اند . آنها با هم مبادلات پول انجام داده اند که در این بین n نفر سود و n نفر ضرر کرده اند . هنگامی که آن ها به خانه هایشان برگشته اند می خواهند کاری کنند که همه بی حساب شوند . هر کس در خانه ی خود پول دارد . در هر مبادله دو نفر شرکت می کنند . کمترین تعداد مبادله که باید انجام شود تا مطمئن باشم همه بی حساب شده اند چقدر است؟
جواب: در هر مبادله می توان یک نفر که سود کرد و یک نفر که ضرر کرده را شرکت دهیم و اگر مقدار ضرر بیشتر باشد کسی که سود کرده همه ی سود خود را به دیگری داده و بی حساب می شود و اگر سور بیشتر باشد کسی که ضرر کرده به اندازه ی ضررش از دیگری پول گرفته و خود را بی حساب می کند . به این سان در هر مبادله حداقل یک نفر بی حساب می شود و در مبادله ی آخر هر دو نفر بی حساب می شوند در نتیجه 2n-1 مبادله انجام می شود .